You are on page 1of 46

Ped Neuro

Ped
CNS
By: Dr. Abdullah Nouh

1
Ped Neuro
EDiR Notes
• Calcifications in tuberous sclerosis are seen in adolescents in periventricular & subependymal locations.
• Fibromatosis coli presents as a mass in sternocleidomastoid muscle for 2 or more weeks after birth due to birth
trauma (forceps, vaginal delivery). Torticollis may be associated.
• Subcortical tubers in tuberous sclerosis generally calcify (but less than the subependymal nodules).
• Fibromatosis coli occur solely within the sternocleidomastoid muscle. Spontaneous regression by 2 years is seen
usually.
• Diffusion sequence has no role in differentiating or diagnosing white matter leukodystrophies.
• Canavan’s disease (involves juxtracortical U fibres) shows diffuse symmetric increased WM signal throughout with
relative sparing of internal & external capsules & corpus callosum. Both globus pallidus show high signal but
sparing of putamen & caudate nuclei are seen. NAA peak is raised spectroscopy & on urine analysis.
• Macrocrania is seen in Canavan’s & Alexander’s.
• In mild to moderate hypoxic brain injury in a full term neonate, there is sparing of brainstem, cerebellum & deep
grey structures.
• Severe hypoxic injury involves ventral & lat thalamus, post putamen, perirolandic cortex of corticospinal tract.
• In adrenoleukodystrophy, there is symmetrical low densities seen in occipito – parieto-temporal WM with thin
curvilinear enhancing rims. There is adrenal gland insufficiency with increased pigmentation & raised ACTH levels.
• In Dandy walker variant, posterior fossa is not enlarged but partial vermian hypoplasia with partial obstruction to
4th ventricle is seen.
• NICE guidelines for children
▪ 3 or more episodes of vomiting.
▪ CSF otorrhoea
▪ (Age > 1yr) - GCS < 14 (In emergency room)
▪ Age < 1yr, paediatric GCS < 15.
▪ Head injury as a result of dangerous mechanism (RTA, fall from 3meteres or projectile injury)
• High riding 3rd ventricle, enlarged foramen of Monro & sunburst gyral pattern with interhemispheric cyst is seen
in agenesis of corpus callosum.
• Choroid plexus is attached to inferomedial aspect of ventricular floor, shows tapering towards caudothalamic groove
& never anterior to foramen of Monro.
• Myelination changes are seen on T1WI till 10 months when it shows similar pattern to adult. Further changes are
assessed on T2WI.
• Normal skull suture width is > 10 mm at length, 3mm at 2 yrs. & 2mm at 3 yrs.
• Hypoxic (severe) damage at term affects putamen, thalami & adjacent WM.
• Brachycephaly is associated with a higher incidence of neurological abnormality composed with scaphocephaly
(more common).
• Sagittal suture is most commonly affected in craniosynostosis.
• Agenesis of corpus callosum shows absence of cavum septum pellucidum, colpocephaly, high riding 3rd ventricle,
widening of interhemispheric fissure & delay in sulcation.
• Abnormal sulcal morphology may cause periventricular nodular heterotopia & parenchymal T2WI signal
hypointensity.
• Brainstem abnormalities are seen in association with cerebellar abnormality.
• Unilateral fusion of lambdoid suture is seen in plagiocephaly (lopsided skull).
• Intrauterine closure of coronal, sugittal & lambdoid suture s/o cloverleaf skull, seen in thanatophoric dysplasia.
• Three patterns of HIE:-
• Periventricular leukomalacia due to prolonged partial asphyxia in preterm or term. It affects watershed
areas, near trigone of lateral ventricles
• Acute profound asphyxia causes lesions in deep grey matter, hippocampus or dorsal brainstem.
• Multicystic encaphalomalacia due to encephalopathy & brain edema.
• Myelination pattern follows caudal to cranial, posterior to anterior & deep to superficial.
• Myelination begins with brainstem & cerebellum, then BG & then peripheral cortical WM.
• Chiari I malformation shows elongated but normal 4th ventricle.
• Chiari II malformation shows caudal displacement of 4th ventricle.
▪ I – AV fistula
▪ II - Angiomatous malformation of BG, thalami & midbrian.
▪ III – Both I & II

2
Ped Neuro
• Cephalhematoma occurs during delivery & shows a crescent shaped firm non pitting parietal mass that does not
cross sutures & lie adjacent to outer table.
• Caput succedaneum is localized scalp edema that crosses suture lines.
• Cephalocele is a skull defect through which meninges, brain & CSF protrude out.
• Germinal matrix haemorrhage in located anterior to caudothalamic groove & inferior to lateral ventricles.
• In mesial temporal sclerosis, asymmetrical atrophy of hippocampus with high T2 signal with atrophy of fornix &
mamillary body is seen. Coronal T2W is required.
• Juvenile angiofibroma are supplied by internal maxillary artery.
• Unmyelinated white matter is bright on T2W & can disguise pathological lesions.
• Oxycephaly is fusion of all cranial sutures & leads to high head. Plagiocephaly is unilateral craniosynostosis.
• NF1 shows renal artery aneurysm & stenosis & abdominal coarctation (Mid aortic syndrome).
Leukodystrophies
Metachromatic U fiber sparing with tigroid pattern due to perivascular sparing
Krabbe’s Thalamic & caudate nuclei high attenuation on CT with U fiber sparing.
MPS Well defined high T2 signal foci in corpus callosum & basal ganglia
X- linked adrenoleukodystrophy Symmetrical peritrigonal & splenium high T2. Peripheral contrast enhancement.
MELAS Multiple cortical& subcortical infarcts like lesion.
Canavan’s U Fibres affected mostly
Alexander Frontal lobe predilection
Subcortical WM affected
• First branchial cleft cyst lies inferior & posterior to tragus.
• Cystic hygroma occurs as cystic mass posterior to SCM in posterior triangle.
• Unilateral cerebral atrophy with leptomeningeal enhancement and developmental venous anomaly is seen in Sturge
weber syndrome.
• The anterior aspects of spinous processes of C1, C2 & C3 should line up within 1 or 2 mm of each other on flexion
& extension.
• Transmantle heterotopia can be confused with closed lip schizencephaly.
• Cervicomedullary kinking is seen both Chiari I & II malformation.
• In Chiari II, vermis herniates through foramen magnum but tonsils lie lateral to medulla. In Chiari I, caudal
displacement of tonsils is seen.
• Bilateral frontal enlargement of subarachnoid space in children is due to benign external hydrocephalus.
• Cafe au lait spots are seen in McCune Albright Syndrome (unilateral FD, precocious puberty & skin problems).
• PVL shows white matter necrosis, multiseptated cystic lesions in periventricular WM with exvacuo dilatation of
ventricles.
• Supratentorial arachnoid cyst occurs in Sylvian fissures.
• Sturge Weber causes angiomatosis in meninges, face & eyes. Retinal angiomas can result in retinal detachment.
• Parallel widely spaced lateral ventricles with dilated trigones & occipital horns with riding 3rd ventricle are signs
of agenesis of corpus callosum.
• Most common cause of congenital hydrocephalus is aqueductal stenosis.
• TS shows subependymal hamartomas, cortical & subcortical tubers & heterotopic grey matter islands in white
matter. These lesions may be calcified.
• Enlarged posterior fossa in not seen in Dandy Walker variant.
• Sphenoid wing hypoplasia is a feature of NF1.
• Hypoplasia of vermis with dilated 4th ventricle is Dandy walker malformation.
• Hydrocephalus & elongation of medulla may be seen in Chiari II malformation as well.
• In Sturge Weber, same sided brain involvement is seen is the port wine stain.
• The falx, corpus collosum & ventricular system can be normal in lobar type of holoprosencephaly but septum
pellucidum is absent in all types.
• Cavum septum pellucidum is a thin CSF density between lateral ventricles & rarely may cause obstructive
hydrocephalus.
• Enlarged medullary/subependymal veins with choroid angiomas are seen in Sturge Weber.
• Enlarged temporal horns is the most sensitive sign of early hydrocephalus on NCCT.
• Communicating hydrocephalus may be seen after 1 wk of SAH when hemorrhage causes impaired CSF absorption.
• Acute obstructive (non communicating) hydrocephalus after SAH (< 7days) is due to intraventricular blood &
resultant ependymitis.
• In NF1, there is hypoplasia of pedicles & posterior elements.

3
Ped Neuro
• Lateral to the lentiform nucleus lies the white matter of external capsule & then the claustrum, a thin sheet of grey
matter. The extreme capsule lies lateral to claustrum & separates it from insular cortex.
• Torcular lambdoid inversion (elevation of torcular herophili above lambdoid suture) is seen in Dandy Walker
malformation.
• Criteria’s for NF: Atleast 2 criteria’s
1. ≥ 6 cafe au lait spots
2. ≥ 2 neurofibromas or one plexiform
3. ≥ 2 lisch nodules
4. Axillary / inguinal freckling
5. Optic glioma
6. Sphenoid dysplasia or thinning of long bone cortex
7. First degree relative with NF1
• In Dandy Walker variant, there is hypoplasia of vermis only with large CSF space surrounding cerebellum.
• Mammillopontine distance is decreased in hydrocephalus.
• Funnel shaped aqueduct is seen in aqueductal stenosis with lack of flow through it on phase contrast MRI. Flow
void sign is seen in normal individuals due to increased CSF velocity through aqueduct.
• Porencephalic cyst is a cystic cavity with adjacent enlargement of lateral ventricle. They develop in utero or early
infancy.
• Adrenoleukodystrophy is posterior white matter involvement with frontal and cerebellar involvement in late stages.
Enhancement of lateral margins can be seen.
• Adrenoleukodystrophy shows marginal enhancement at the anterior or lateral aspect of the lesion.
• Lindau tumours are retinal angiomas, hemangiomas & hamartomas seen in VHL. They can cause calcification.
• (CNS) Fibromatosis coli (pseudotumour of SCM muscle) occurs in first 2 - 4 wks of life, associated with breech or
forceps delivery. Firm, non-tender mass is seen in lower 2/3 of SCM. Resolves by 1st year of life.

4
Ped Neuro
Succeeding
2. In mild-moderate hypoxic brain injury in a full term neonate typically spares:
A. Watershed areas
B. Parasagital cortex
C. Deep grey matter structure
D. Subcortical white matter
E. Frontal lobes

2. C
The brainstem, cerebellum and deep grey structures are spared in mild-moderate ischemia. Severe hypoxic injury
involves the ventral and lateral thalamus, posterior putamen, perirolandic regions and corticospinal tracts.
3. A 7-year-old boy with a well mother and father develops a progressive deterioration in vision, loss of hearing, optic
disc pallor and ataxia. Mental deterioration is also noted. CT shows symmetric low densities in the occipitoparietal
temporal white matter with thin curvilinear enhancing rims. MR shows hypodensity on T1 and bilateral hyperintense
confluent areas. Which is the most likely diagnosis?
A. Adrenomyloneuropathy
B. Adrenoleukodystrophy
C. Friedreich’s ataxia
D. Ataxia telangiectasia
E. Spongiform leukodystrophy

3. B
Features describe adrenolukodystrophy, an X linked recessive condition. Patients also have adrenal gland insufficiency
with increased pigmentation and raised Adrenocorticotropic Hormone (ACTH) levels.
4. The absence of which of the following indicates a diagnosis of Dandy-Walker variant rather than Dandy-Walker
malformation?
A. Dysgenesis of the corpus callosum
B. Holoprosencephaly
C. Cerebellar heterotopia
D. Enlargement of the posterior fossa
E. Cerebellar gyri malformation

4. D
The other features are common to both.
(CNS) 8. A CT brain of a 25-year-old male with a head injury but no focal neurology shows no acute abnormality. A
thin cerebrospinal fluid (CSF) density is noted between the frontal horns of the lateral ventricles in the midline. Which
is the diagnosis?
A. Cavum septi pellucidi
B. Cavum vergae
C. Cavum veli interpositii
D. Colloid cyst
E. Arachnoid cyst

8. A
Occurs in 80% of term infants and 15% of adults. Rarely may dilate and cause obstructive hydrocephalus.
12. MRI brain in a neonate shows multiple abnormalities including an anterior interhemispheric fissure adjoining a high
riding third ventricle, an enlarged foramen of monro and a sunburst gyral pattern. An interhemispheric cyst is also seen.
Which is the diagnosis?
A. Arachnoid cyst
B. Agenesis of the corpus callosum
C. Prominent cavum septum pellucidi and vergae
D. Chiari II malformation
E. Dandy-Walker malformation

12. B
Absence of septum pellucidum, corpus callosum and cavum septum pellucidi, and wide separation of the lateral
ventricles, are other features of agenesis of the corpus callosum.

5
Ped Neuro
14. Ovoid areas of increased echogenicity are seen on US of a neonate. Which of the following favours choroid plexus
rather than germinal mature haemorrhage?
A. Tapering toward cardiothalamic grove
B. Inferolateral to floor of frontal horn
C. Bulbous enlargement of cardiothalamic groove
D. Location anterior to foramen of Monro
E. Low birth weight and premature neonate

14. A
Prematurity and low birth weight are amongst risk factors for GMH. Chord plexus is attached to inferomedial aspect of
ventricular floor, tapering toward cardiothalamic groove and never anterior to the foramen of Monro.

(CNS) 17. Which of the following associations favour Chiari I rather than Chiari II malformation?
A. Klippel-Feil anomaly
B. Myelomyeocele
C. Dysgenesis corpus callosum
D. Absence of septum pellucidi
E. Excessive cortical gyration

17. A
Syringohydromyelia, hydrocephalus, malformation of small bones and spine are also associations of Type I Chiari
malformation.

(CNS) 25. A combination of subependymal nodules, giant cell astrocytomas, white matter lesion and retinal
phakomatoses suggests:
A. Tuberous sclerosis
B. NFI
C. NF2
D. Sturge-Weber syndrome
E. Von Hippel-Lindau

25. A
These are all features of tuberous sclerosis.

(CNS) 31. A patient with a genetic condition and known bilateral renal cysts undergoes MRI brain showing a cystic
lesion with mural nodule. This finding is associated with:
A. VHL
B. Tuberous sclerosis
C. Sturge-Weber
D. Heritary Haemorrhagic Telangiectasia (HHT)
E. Neurofibromatosis

31. A
1/3 haemangioblastomas are purely solid, 1/3 are cystic with mural nodes and 1/3 are cystic with a more complex solid
component.

(CNS) 32. Tram track gyriform calcification is most likely to be seen in:
A. NF I
B. Tuberous sclerosis
C. Von Hippel-Lindau
D. HHT
E. Sturge-Weber

32. E
Other features include cerebral atrophy with thickening of skull vault and surface enhancement, enlarged choroid plexus
and enlarged medullary/subependymal veins. Choroid angiomas may be present.

6
Ped Neuro
Get through
1) A finding of bilateral meningiomas in a child suggests a diagnosis of which of the following phakomatoses?
a. neurofibromatosis type 1
b. neurofibromatosis type 2
c. von Hippel-Lindau disease
d. Sturge-Weber syndrome
e. tuberous sclerosis

1) b. ___
The phakomatoses are neuroectodermal disorders with skin and central nervous system manifestations.
Neurofibromatosis type 2 (central neurofibromatosis) accounts for only 10% of cases, type 1 accounting for 90%.
Although the hallmark finding of type 2 is bilateral acoustic schwannomas, multiple meningiomas, particularly in a
child, should bring the diagnosis to mind. A mnemonic for remembering the associations is MISME (multiple inherited
schwannomas, meningiomas and ependymomas), and tumours may be seen in the brain or spinal cord. In the spine,
multiple nerve sheath tumours are often located in the cauda equina. Skin manifestations are seen relatively infrequently,
unlike in the more common type 1.
(CNS) 2) A child who undergoes MR of the brain for clinically apparent facial abnormalities is shown to have a defect
of midline cleavage of the brain. What structure is abnormal or absent in all forms of holoprosencephaly, and therefore
is the most sensitive indicator of a midline cleavage abnormality?
a. falx cerebri
b. third ventricle
c. fourth ventricle
d. corpus callosum
e. septum pellucidum

2) e. ___
Holoprosencephaly is failure of the primitive brain to cleave into two hemispheres, and is commonly associated with
midline facial abnormalities (ranging from cyclopia to hypertelorism) and absence of many intracranial midline
structures. There are three types, the most severe being the alobar form, which shows no cleavage at all, with absence
of the falx cerebri and third ventricle, fusion of the cerebral hemispheres and thalami, and a single large lateral ventricle.
The semilobar form has variable cleavage with a partially formed falx, rudimentary third ventricle, and variable cleavage
of the thalami, lateral ventricles and cerebral hemispheres. In the lobar type of holoprosencephaly, brain formation may
be nearly normal, but the septum pellucidum is always absent, as in all forms. The falx, corpus callosum and ventricular
system may be normal in the lobar type.
6) A 6-year-old boy presents with learning disability, seizures and a facial rash. MRI of the brain shows several cortical
lesions with low signal on T1W and high signal on T2W images. They show no enhancement with administration of
intravenous gadolinium. Low-signal subependymal nodules are identified on all sequences. Subsequent renal ultrasound
scan demonstrates multiple, bilateral hyperechogenic lesions. What is the most likely diagnosis?
a. neurofibromatosis type 1
b. Sturge-Weber syndrome
c. von Hippel-Lindau disease
d. tuberous sclerosis
e. metachromatic leukodystrophy

d. ___
Tuberous sclerosis is a member of the phakomatoses, a group of neuroectodermal disorders characterized by coexistence
of skin and central nervous system tumours. The classic clinical triad of seizures, learning disability and adenoma
sebaceum (a facial rash) is seen in approximately half of presenting patients. Imaging findings seen on MRI and
sometimes CT are periventricular nodules, which often calcify, and ‘cortical tubers’, which represent brain parenchymal
hamartomas. These lesions are low signal on T1 and high signal on T2W images, and typically do not enhance.
Enhancement of a lesion suggests associated giant cell astrocytoma, which is most commonly seen at the foramen
ofMonro, where it may obstruct, causing hydrocephalus. Common associations are renal angiomyolipoma, bone
abnormalities and spontaneous pneumothorax.

7
Ped Neuro
(CNS) 22) What is the most sensitive sign on non-contrast CT for detecting early hydrocephalus?
a. cortical sulcal effacement
b. uncal herniation
c. enlarged third ventricle
d. enlarged fourth ventricle
e. enlarged temporal horns of the lateral ventricles

22) e. ___
In many cases of hydrocephalus due to subarachnoid haemorrhage, the temporal horns of the lateral ventricles become
dilated sooner than the frontal horns. Dilatation of the temporal horn is often particularly conspicuous, as it is frequently
not visualized at all on CTof normal brains. Uncal herniation is herniation of the medial temporal lobe into a subtentorial
location, where it may exert pressure on the brain stem and is a late sign of raised intracranial pressure, often presenting
with oculomotor nerve palsy resulting in a fixed dilated pupil.

23) MRI of the brain in a premature baby reveals ischaemic lesions adjacent to the trigone of the lateral ventricle. What
is the most likely insult to have caused these appearances?
a. prolonged partial asphyxia
b. acute profound asphyxia
c. germinal matrix haemorrhage
d. rupture of a choroid plexus cyst
e. venous sinus thrombosis

23) a. ____
There are three patterns of hypoxic ischaemic encephalopathy. Periventricular leukomalacia occurs in watershed areas
of arterial distribution. It is caused by prolonged partial asphyxia in preterm or term babies. Acute profound asphyxia
causes lesions in the deep grey matter, hippocampus and dorsal brain stem. Lastly, there is multicystic encephalomalacia
that follows devastating encephalopathy and generalized brain oedema.

(CNS) 27) A 22-year-old man with von Hippel-Lindau syndrome presents with headaches, vomiting and ataxia. CT of
the brain demonstrates an abnormality in the posterior fossa. What are the most likely findings?
a. cystic lesion with an enhancing mural nodule
b. multiple ring-enhancing lesions with surrounding oedema
c. gyriform cortical calcifications
d. multiple calcified subependymal nodules
e. hypodense, ill-defined mass with central necrosis and marked surrounding oedema

27) a. ___
Haemangioblastomas are the most commonly recognized manifestation of von Hippel-Lindau syndrome. They usually
occur in the cerebellum and may be multiple in up to 15% of cases. Patients also commonly develop renal cell carcinoma,
and difficulty may occasionally arise in distinguishing metastases from multiple haemangioblastomas. Typical features
are of a cystic mass in the hemisphere or vermis, with an enhancing mural nodule, though entirely solid lesions may
also occur. Calcification is not a feature. Gyriform cortical calcifications are a feature of Sturge-Weber syndrome, and
usually occur in the temporoparieto- occipital region. Calcified subependymal nodules (hamartomas) are a feature of
tuberous sclerosis. A hypodense, ill-defined mass with central necrosis and marked surrounding oedema is a classic
appearance of a glioblastoma multiforme.

32) Of the choices below, which best describes the pattern of normal myelination of the brain in the first 9 months of
life?
a. cranial to caudal, posterior to anterior, deep to superficial
b. cranial to caudal, anterior to posterior, deep to superficial
c. cranial to caudal, posterior to anterior, superficial to deep
d. caudal to cranial, posterior to anterior, deep to superficial
e. caudal to cranial, anterior to posterior, superficial to deep

8
Ped Neuro
32) d. ____
Myelination is an important feature of the maturation of the normal central nervous system. It is a dynamic process that
begins in utero and continues after birth in a predetermined manner. It is well demonstrated on MRI, where initially
white and grey matter show the reverse signal characteristics to those seen in the adult brain, with the white matter
appearing of lower signal on T1 and higher signal on T2 than grey matter.
As myelination occurs, the white matter gains fat content and so becomes of higher signal on T1 and lower signal on T2
than grey matter, with completion at around 9 months. The process progresses caudal to cranial, posterior to anterior,
and deep to superficial, beginning with the brain stem and cerebellum, then the basal ganglia, with the final areas to
mature being the peripheral cortical white matter.

(CNS) 32) An 18-month-old toddler presents with progressive gait disturbance, motor developmental delay and muscle
weakness. Biochemical tests show abnormally low levels of arylsulphatase A enzyme in peripheral blood leukocytes
and urine. MRI demonstrates symmetrical, confluent areas of high signal on T2W images affecting the periventricular
and cerebellar white matter, corpus callosum and corticospinal tracts with sparing of the subcortical U fibres. The
periventricular abnormality shows a striped ‘tigroid’ pattern. There is no enhancement with administration of
intravenous gadolinium. What is the most likely condition?
a. Alexander’s disease
b. Canavan’s disease
c. acute disseminated encephalomyelitis
d. mucopolysaccharidosis
e. metachromatic leukodystrophy

32) e. ____
Dysmyelinating diseases (leukodystrophies) are a wide spectrum of inherited neurodegenerative disorders affecting
myelin in the brain and peripheral nerves. Most fall into one of three categories: lysosomal storage diseases, peroxisomal
disorders and diseases caused by mitochondrial dysfunction. The most common, metachromatic leukodystrophy, is an
autosomal recessive disorder caused by a deficiency of the lysosomal enzyme arylsulphatase A, which leads to
accumulation of sulphatides in tissues. It usually manifests as a late infantile subtype in children at 12–18 months of
age, and is characterized by motor signs of peripheral neuropathy followed by deterioration in intellect, speech and
coordination, leading to death within a few years. On T2W MRI, symmetrical confluent areas of high signal intensity
are seen in the periventricular white matter with sparing of the subcortical U fibres. Sparing of the perivascular white
matter gives a striped or tigroid appearance to the periventricular area of abnormality, particularly well seen in the
centrum semiovale. Other sites often affected are the corpus callosum, internal capsule and corticospinal tracts.

(CNS) 35) A 9-month-old boy presents to the paediatric neurologist with general developmental delay, left-sided
weakness from birth and tonic-clonic seizures. MRI performed under anaesthetic reveals a thin cleft containing CSF
extending from the right lateral ventricle to the cortical surface of the right frontal lobe. The margins of the cleft are
opposed and lined with heterotopic grey matter that shows polymicrogyria. Which disorder of cortical formation is
described?
a. porencephaly
b. schizencephaly
c. holoprosencephaly
d. lissencephaly
e. hemimegalencephaly

35) b. __
The development of the cerebral cortex takes place in three stages: cell proliferation, cell migration and cortical
organization. Schizencephaly is a disorder of the final stage where there is a CSF-containing cleft lined with grey matter
(which is often polymicrogyric) connecting the subarachnoid CSF space with the ventricular system. Differentiation can
be made from the cyst or cavity of porencephaly, by the presence in schizencephaly of a lining of heterotopic grey
matter, whereas a porencephalic cyst is lined by white matter. To result in schizencephaly, the insult must involve the
entire thickness of the cerebral hemisphere during cortical organization, as in injuries due to prenatal infection,
ischaemia or chromosomal abnormalities. Clinical presentation varies but often includes developmental delay, motor
impairment and seizures.

9
Ped Neuro
51) A 12-year-old girl presents with altered sensation in the upper and lower limbs. Clinical assessment demonstrates
weakness of the lower limbs with reduced pain and temperature sensation. MRI shows syringohydromyelia and tonsillar
ectopia of 7 mm with the fourth ventricle in normal position. No myelomeningocele is seen. What is the most likely
diagnosis?
a. Chiari I malformation
b. Chiari II (Arnold-Chiari) malformation
c. Chiari III malformation
d. Dandy-Walker syndrome
e. diastematomyelia

51) a. ___
Chiari I malformation is characterized by tonsillar ectopia. The fourth ventricle is elongated but normal in position. It is
associated with syringohydromyelia, hydrocephalus and malformations of the skull base. Chiari II (Arnold-Chiari)
malformation is characterized by hindbrain abnormalities, with caudal displacement of the fourth ventricle. A lumbar
myelomeningocele is seen in over 95% of cases. Chiari III malformation is rare and has a high cervical/low occipital
meningomyelo-encephalocele. Survival beyond infancy is unusual. In Dandy-Walker syndrome, there is enlargement
of the posterior fossa with cystic dilatation of the fourth ventricle and abnormalities of the cerebellar vermis.
Diastematomyelia results in sagittal splitting of the spinal cord into two hemi-cords, and is sometimes associated with a
myelomeningocele.
(CNS) 63) A 20-year-old male patient has an MR scan of his spine for investigation of back pain. He has low IQ and
multiple skin patches. The MR scan shows bilateral branching tubular paraspinal masses, with widening of the
intervertebral foramina and posterior scalloping of the vertebral bodies. A sharply angulated kyphosis is present at the
thoracolumbar junction. What is the most likely diagnosis?
a. neurofibromatosis type 1
b. neurofibromatosis type 2
c. tuberous sclerosis
d. Marfan’s syndrome
e. Ehlers-Danlos syndrome

63) a. __
Neurofibromatosis type 1 (peripheral neurofibromatosis or von Recklinghausen’s disease) is a multisystem disorder
affecting the majority of organ systems. The presence of plexiform neurofibroma is pathognomonic. In the spine, there
is abnormal development of the vertebral bodies with hypoplasia of pedicles and posterior elements. Dural ectasia is
seen secondary to weakness of the meninges. Neurofibromatosis type 2 (central neurofibromatosis) is characterized by
bilateral acoustic neuromas, meningiomas and ependymomas. Tuberous sclerosis produces multiple hamartomas and
malformations of several organ systems. Marfan’s and Ehlers-Danlos syndromes may also cause posterior scalloping of
vertebral bodies.
(CNS) 65) A severely hypoplastic cerebellar vermis in an enlarged bony posterior fossa, with associated hydrocephalus
and communication of the fourth ventricle with a posterior midline CSF cyst, are features of which of the following
posterior fossa malformations?
a. mega cisterna magna
b. Dandy-Walker malformation
c. Dandy-Walker variant
d. Arnold-Chiari malformation
e. Joubert’s syndrome

65) b. __
Classically, the Dandy-Walker malformation consists of partial or total absence of the cerebellar vermis, dilatation of
the fourth ventricle into a large cystic mass, an enlarged posterior fossa, hydrocephalus (in 75% of cases) and torcular-
lambdoid inversion (elevation of the torcular herophili above the lambdoid suture). The proposed aetiology is
obstruction of CSF outflow at the foramina of Magendie and Luschka. The vermis abnormality is the key component in
all forms of the Dandy-Walker complex. The variant is less severe with a better prognosis. Chiari malformations have
the fundamental abnormality of an underdeveloped, small posterior fossa, in contrast to the Dandy- Walker complex
where it is normal or enlarged.
4

10
Ped Neuro
66) During a routine new-baby check, a unilateral, firm, tense, nonpitting, parietal mass is noticed. Ultrasound scan
demonstrates a crescent-shaped lesion adjacent to the outer table of the skull. The mass is most likely to be which of the
following?
a. caput succedaneum
b. cephalocele
c. cephalhaematoma
d. leptomeningeal cyst
e. fibrous dysplasia

66) c. ___
Cephalhaematoma is seen with birth trauma, particularly following poor instrumentation and skull fracture during delivery. It is
seen in 1-2% of deliveries. It can grow after birth and takes weeks or months to resolve. It does not cross sutural lines because the
haematoma is beneath the outer layer of periosteum. The haematoma can calcify. Caput succedaneum is localized scalp oedema that
does cross sutural lines. A cephalocele is a skull defect through which meninges, brain and cerebrospinal fluid may protrude.
67) Hyperechoic lesions are seen within the brain of a preterm neonate during a cranial ultrasound scan. Of the following,
which most strongly suggests that the appearances are due to germinal matrix haemorrhage rather than periventricular
leukomalacia?
a. the baby is premature
b. the hyperechoic changes are anterior to the caudothalamic groove
c. the hyperechoic changes are adjacent to the trigone of the lateral ventricle
d. cystic changes follow the acute phase in the affected brain
e. there was no birth trauma

67) b. ____
Germinal matrix is highly vascular tissue seen at 24-32 weeks’ gestational age, located anterior to the caudothalamic
groove and inferior to the lateral ventricles. It is at risk of hypoxaemia and ischaemia. Haemorrhage here can be
promoted by trauma at birth or coagulopathy including rhesus incompatibility. Germinal matrix haemorrhage less than
7 days old is hyperechoic but without shadowing. Within 2-3 weeks, the abnormal area decreases in size and the
echogenicity reduces. Periventricular leukomalacia is white matter necrosis following hypoxaemia. It is seen in 5-10%
of preterm babies. It occurs in arterial watershed areas and may appear acutely as a broad region of periventricular
increased echogenicity. Cystic degeneration may follow after 2 weeks or more.

70) A 15-year-old girl with a history of multiple febrile convulsions as an infant presents to the neurologist with complex
partial seizures. The episodes start with special and somatosensory aura followed by a wide-eyed stare with behavioural
arrest, before finally proceeding to a generalized tonic-clonic seizure. MR imaging is requested. Which sequence is best
to evaluate the temporal lobes for signs of mesial temporal sclerosis?
a. axial T1W
b. axial T2W
c. axial FLAIR
d. coronal T1W
e. coronal T2W

70) e. ____
Mesial temporal sclerosis is a pattern of hippocampal neuronal loss that can occur with long-standing temporal lobe
epilepsy as a result of excessive neuronal depolarization leading to cytotoxic oedema. Three patterns of loss are
described, with relative sparing of the CA2 subfield often a feature. The typical findings are of asymmetrical atrophy of
the hippocampus with abnormally high signal returned on T2W images. Ipsilateral findings in the limbic system include
atrophy of the fornix and maxillary body. Cortical abnormalities may exist in the temporal cortex, and this is best
evaluated on high-resolution T1 sequences.

78) Germinal matrix haemorrhage is identified on a cranial ultrasound scan of a neonate. Which of the following imaging
features confers the worst prognosis?
a. subependymal haemorrhage
b. intraventricular haemorrhage
c. intraventricular haemorrhage with ventricular dilatation
d. intraparenchymal haemorrhage
e. choroid plexus pulsation

11
Ped Neuro
78) d. ___
Subependymal haemorrhage (grade 1) usually has no long-term consequence. Mortality rates for intraventricular
haemorrhage, intraventricular haemorrhage with ventricular dilatation and intraparenchymal haemorrhage are 10%,
20% and more than 50%, respectively. These represent grades 2, 3 and 4 haemorrhage. Normal choroid plexus pulsates
while haematoma at the same location is not pulsatile.
82 A term neonate whose birth per vaginum was notably protracted has an MRI of the brain at age 4 days. There are
changes in keeping with hypoxic ischaemic encephalopathy. Affected areas are marked by T1 shortening without T2
signal change. Which of the following is the most likely explanation for this pattern of abnormal signal?
a. high T2 signal is masked by a high degree of myelination
b. high T2 signal is masked by a low degree of myelination
c. high T1 signal is accentuated by a high degree of myelination
d. high T1 signal is accentuated by a low degree of myelination
e. low T1 signal is masked by a high degree of myelination

83) b. ____
Neonatal hypoxic ischaemic encephalopathy occurs in 1-2/1000 live births. Clinically, it manifests as disturbed
neurological function such as difficulty with respiration, abnormal tone, depressed reflexes, altered level of
consciousness, feeding difficulties or seizures. Initial MRI findings, particularly in the first week of life, can be T1
shortening (bright) rather than a high T2 signal. This is because the unmyelinated white matter is brighter on T2W
images and can disguise pathological lesions. T1 shortening is thought to be due to lipid breakdown products of damaged
myelin or to mineralization.

85) Craniosynostosis of the sagittal suture of the skull results in which skull vault shape abnormality?
a. brachycephaly
b. scaphocephaly
c. trigonocephaly
d. oxycephaly
e. plagiocephaly

85) b. ___
Craniosynostosis means premature closure of a suture of the skull, and skull growth is arrested in the direction
perpendicular to the affected suture. Craniosynostosis of the sagittal suture therefore results in a head shape that is
abnormally long and narrow and has the appearance of an upturned boat. This is called scaphocephaly (or
dolichocephaly) and accounts for 60% of all craniosynostoses. Closure of the coronal suture accounts for 20% and
results in a short, wide head called brachycephaly. Trigonocephaly is caused by craniosynostosis of the metopic suture
and results in a forward-pointing head. Oxycephaly is the most extreme form, and all sutures may be affected, resulting
in a high head. Plagiocephaly is unilateral craniosynostosis. Early recognition of all types is important to prevent
permanent deformity and allow for surgical correction. Causes may be primary or secondary to dysplasia, a number of
genetic syndromes, microcephaly, and metabolic or haematological conditions.

(Ped) 88) From the following renal and adrenal findings, choose that which favours a diagnosis of neurofibromatosis
type 1 above von Hippel–Lindau syndrome or tuberous sclerosis.
a. renal cysts
b. renal cell carcinoma
c. phaeochromocytoma
d. renal artery aneurysm
e. angiomyolipoma

88) d. __
Renal artery stenosis and aneurysms plus abdominal coarctation are associated with neurofibromatosis type 1 (NF1).
Renal cysts are found in von Hippel–Lindau syndrome (VHL) and tuberous sclerosis, while renal cell carcinoma is
found in VHL. NF1 and VHL are associated with phaeochromocytoma. Angiomyolipomas, usually multiple and
bilateral, are found in 50% of cases of tuberous sclerosis.

12
Ped Neuro
89) A 12 year old presents with a painless swelling in the right side of his neck, at the angle of the mandible. Ultrasound
scan shows a cystic lesion with internal debris. CT confirms a cystic lesion with a beak of tissue pointing between the
internal and external carotid arteries. What is the most likely diagnosis?
a. cystic hygroma
b. abscess
c. branchial cleft cyst
d. carotid body tumour
e. lymphadenopathy

89) c. ___
The described features are typical of a branchial cleft cyst arising from the second branchial cleft. The mass displaces
the sternocleidomastoid posteriorly, and the carotid and internal jugular vessels posteromedially. The pointing between
the internal and external carotid arteries is pathognomonic. Cystic hygromas present in infancy as a cystic mass in the
posterior cervical space. Abscesses present with associated symptoms and a painful red swelling, though imaging
findings may be similar. Carotid body tumours are solid, highly vascular masses lying between the internal and external
carotid artery origins.
Barret1
2 A 15 year old boy is under investigation for hypertension. The serum catecholamines and urinary VMAs are raised.
CT abdomen confirms a left adrenal phaeochromocytoma and additionally shows multiple cysts within the liver,
pancreas and both kidneys. Which of following additional features would you look for?
(a) Bilateral acoustic neuromas
(b) CNS haemangioblastomas
(c) Iris hamartomas
(d) Parathyroid adenomas
(e) Pulmonary lymphangioleiomyomatosis

2 (b)
VHL is associated with CNS haemangioblastomas in 50%, pancreatic/ hepatic/ renal cysts, RCC, and
phaeochromocytoma (10%). Phaeochromocytomas are also associated with MEN types 2A and 2B (also medullary
thyroid cancer, parathyroid adenomas), NF-1, & TS. Bilateral acoustic neuromas are associated with NF-2.
(CNS) 5 A young patient with learning difficulties presents with multiple skin lesions and seizures. On examination
there is an erythematous facial rash in a butterfly distribution and subungual fibromas are noted in the hands. A CT head
is performed. Which of the following is not typically associated with this condition?
(a) Sclerotic lesions in the skull
(b) Asymptomatic, calcified retinal nodules
(c) Hydrocephalus
(d) Enhancing, calcified cortical tubers
(e) Cerebral arterial ectasia

5 (d)
The CNS features of tuberous sclerosis include: subependymal nodules which increase in number and calcification with
time; parenchymal tubers which are non-enhancing and rarely calcified; hydrocephalus; giant cell astrocytomas; retinal
phakomas; arterial ectasia.
(CNS) 6 A newborn child suffers respiratory distress, bradycardia and spasticity. An MRI of the brain is performed.
Which of the following features is not associated with a Chiari II malformation?
(a) Small posterior fossa
(b) Towering cerebellum
(c) Small 4th ventricle
(d) Hyperplastic falx
(e) Stenogyria

6 (d)
A complex of anomalies secondary to a small posterior fossa. The falx is hypoplastic/ fenestrated, with concomitant
gyral interdigitation. A summary of the major features: the 4th ventricle and brainstem are caudally displaced, there is
tonsillar herniation, a myelomenigiocoele, syringohydromelia, dygenesis of the corpus callosum, obstructive
hydrocephalus, absence of the septum pellucidum, and stenogyria.

13
Ped Neuro
16 Which of the following types of craniosynostosis is correctly linked to the suture involved?
(a) Anterior plagiocephaly - unilateral lambdoidal suture
(b) Brachycephaly - metoptic suture
(c) Posterior plagiocephaly - unilateral coronal suture
(d) Scaphocephaly - sagittal suture
(e) Trigonocephaly - bilateral coronal suture

16 (d)
The most common single suture affected is the sagittal suture (scaphocephaly), others include uni-coronal (anterior
plagiocephaly), bi-coronal (brachycephaly), metoptic (trigonocephaly) and unilateral lambdoidal (posterior
plagiocephaly). Craniosynostosis may be part of a syndrome, with multiple sutures involved, or secondary to drugs,
metabolic disease, or other causes of microcephaly.
28 A 3 year old boy presents with a first seizure; he is afebrile. CT scan shows calcification within the right parietal gyri
with ipsilateral skull thickening and enlargement of the choroid plexus. What is the likely diagnosis?
(a) Klippel-Trenaunay syndrome
(b) Neurofibromatosis type 2
(c) Sturge-Weber syndrome
(d) Tuberous sclerosis
(e) von Hippel Lindau syndrome

28 (c)
Sturge-Weber syndrome is associated with a 'port-wine stain'' naevus in the distribution of the trigeminal nerve, tram-track gyral
calcification (usually parietal lobe), ipsilateral choroid plexus enlargement, leptomeningeal venous angiomas, hemiparesis, seizures,
mental retardation, glaucoma (30%) and choidoidal haemangiomas of the orbit.
(CNS) 31 A CT head demonstrates cerebellar vermian hypoplqsia, relatively normal cerebellar hemispheres, and a large
CSF space surrounding the cerebellum. The brainstem is not disordered. What is the diagnosis?
(a) Dandy Walker complex
(b) Dandy Walker variant
(c) Mega cisterna magna
(d) Chiari Type II
(e) Spenoidal encephalocoele

31 (b)
The above is a description of Dandy Walker variant. By contrast, in Dandy Walker complex the cerebellar hemispheres
and vermis are both absent or hypoplastic and the brainstem is compressed by a cyst. In mega cisterna magna, the
cerebellum is normal and there is a simple dilatation of the CSF space posterior to the cerebellum.

(CNS) 34 An elderly man is found collapsed, and a CT head is performed. The ventricles are noted to be prominent.
Which of the following features would support the diagnosis of hydrocephalus over cerebral atrophy?
(a) Concave profile of the third ventricle on axial images
(b) Enlarged choroidal-hippocampal fissures
(c) Normal fornix-corpus callosum distance
(d) Atrophy of the corpus callosum
(e) Mammillo-pontine distance of less than 1 cm

34 (e)
Indicators of hydrocephalus include: dilatation of the recesses of the third ventricle, convexity of the third ventricle,
expansion of the temporal horns, effacement of sulci, narrowing of the mamillopontine distance, and enlargement of the
ventricles out of proportion to the sulcal dilatation. Specific MR findings include: transependymal CSF exudation (seen
as high signal on FLAIR imaging), and accentuation of the aqueductal flow void in normal pressure hydrocephalus.
38 A 5 year old girl presents with a swelling on the left side of her neck. An MRI examination is subsequently arranged
for investigation. Which feature makes branchial cleft cyst more likely than cystic hygroma?
(a) Associated coarctation of the aorta
(b) Extending between ECA/ICA
(c) Extension into mediastinum
(d) High signal on T2W MR imaging
(e) Multi-loculated appearance on US

14
Ped Neuro
38 (b)
Branchial cleft cyst is usually in the anterior triangle. The 'beak sign', where tissue points between the ICA and ECA is
described as pathognomonic. Cystic hygroma is a single or multiloculated lymphangioma and is associated with
chromosomal anomalies, including Turner's syndrome (coarctation in 15%). Both are high signal on T2, cystic hygroma
may be low or high signal on T1 (depending on levels of lipid/ protein/ blood products).
40 Which of the following causes of neonatal hydrocephalus is more likely to be communicating rather than
noncommunicating?
(a) Aqueduct stenosis
(b) Arnold-Chiari syndrome
(c) Dandy-Walker syndrome
(d) Encephalocoele
(e) Germinal matrix haemorrhage

40 (e)
Obstructive (non-communicating) causes are more common and include spina bifida, aqueduct stenosis, Dandy-Walker syndrome,
Arnold-Chiari syndrome, meningocoele, and encephalocoele. Nonobstructive causes result from reduced reabsorption within the
arachnoid granulation tissue, e.g. infection or haemorrhage, or increased CSF production e.g. choroid plexus papilloma; these can
also cause obstructive hydrocephalus also.
41 A request is made for an MRI examination in a neonate. An antenatal US had revealed agenesis of the corpus
callosum. The MRI scan shows an abnormal posterior fossa. Which of these features make Dandy-Walker a more likely
diagnosis than Chiari II malformation?
(a) Hydrocephalus
(b) Klippel-Feil anomaly
(c) Large posterior fossa
(d) Syringohydromyelia
(e) Towering' cerebellum

41 (c)
Dandy-Walker malformation results from congenital atresia of the foramina of Magendie and Luschka, leading to an
enlarged posterior fossa, a large posterior fossa cyst, hydrocephalus and varying degrees of cerebellar hemisphere and
verminian hypoplasia. Both have syringohydromyelia and hydrocephalus; Chiari II has a small posterior fossa; Chiari I
is associated with Klippel-Feil.

(CNS) 41 A patient presents with suspected neurofibromatosis type 1. Of the following lists of features, which would
not be sufficient to make the diagnosis?
(a) 6 cafe-au-lait macules and two neurofibromas
(b) A Lisch nodule and an optic nerve glioma
(c) Thinning of long bone cortex and axillary freckling
(d) A first-degree relative with NF-1 and inguinal freckling
(e) A plexiform neurofibroma and a sphenoid dysplasia

41 (c)
At least two of the following seven criteria must be fulfilled for a diagnosis: > 6 cafe-au-lait spots, > 2 neurofibromas
of any type (or one plexiform neurofibroma), freckling of the axillary or inguinal region, optic glioma, > 2 Lisch nodules,
a distinctive osseous lesion such as sphenoid dysplasia or thinning of long bone cortex, a 1st -degree relative with NF-1.
(GU) 43 A 35 year old man with a history of ataxia presents unconscious after a fall. A previous unenhanced CT
abdomen demonstrated multiple solid lesions within the right kidney, an absent left kidney with surgical clips nearby
and multiple well-defined cystic lesions within the pancreas. What is the most likely diagnosis?
(a) Tuberous sclerosis
(b) Amyloidosis
(c) Von Hippe! Lindau syndrome
(d) Neurofibromatosis type 1
(e) Lymphoma

43 (c)
VHL disease is an autosomal dominant neurocutaneous syndrome characterised by renal cell carcinomas, often multiple
and bilateral along with cystic renal and pancreatic disease, phaeochromocytomas and spinal, cerebellar and optic nerve
haemangioblastomas. The cerebellar lesions can present as ataxia.

15
Ped Neuro
Barret2
(GU) 6 A 34 year old man presents with gradual onset bilateral loin pain. Ultrasound demonstrates multiple bilateral
renal cystic masses. Which of the following would not support a diagnosis of VonHippel Lindau?
(a) Contrast enhancement on CT of more than 20 HU
(b) Raised urinary catecholamines
(c) Multiple pancreatic cysts
(d) Cutaneous angiofibromas
(e) Cerebellcir haemangioblastomas on brain MRI

6 (d)
VHL is associated with CNS, haemangioblastomas, retinal angiomas, pancreatic cysts and carcinomas and
phaeochromocytoma. In the kidney it is associated with the presence of RCC and multiple cysts. The renal cell tumours
are usually of the clear cell type, and have a variable appearance depending on the degree of soft tissue involvement.
Cutaneous angiofibromas are seen in tuberous sclerosis.

(CNS) 9 A plain radiograph is performed on a male child. Unilateral, premature fusion of both the coronal and lambdoid
sutures is evident. What is the most appropriate description?
(a) Scaphocephaly
(b) Brachycephaly
(c) Plagiocephaly
(d) Trigonocophaly
(e) Oxycephaly

9(c)
The common craniosynostoses include:
Craniosynostosis type Sutures prematurely fused
Scaphocephaly Sagittal
Brachycephaly Coronal or lambdoid
(but not both; can be bilateral)
Plagiocephaly Coronal and lambdoid
Trigonocephaly Metopic
Oxycephaly Sagittal, coronal and lambdoid
Cloverleaf Multiple paired sutures to produce a 'trilobar skull'
11 A cranial US is performed in a pre-term neonate. There is hyperechoic material within the ventricles consistent with
recent haemorrhage, but the ventricles are not dilated. How would you grade this germinal matrix bleed?
(a) Grade I
(b) Grade II
(c) Grade Ill
(d) Grade IV
(e) Grade V

11 (b)
Grade I: subependymal haemorrhage,
Grade II: intraventricular haemorrhage, no ventricular dilation (10% mortality),
Grade Ill: intraventricular hemorrhage with ventricular dilation (20% mortality),
Grade ·1v: intraparenchymal haemorrhage (>50% mortality). There is no Grade V.
(CNS) 12 A patient develops dissociated anaesthesia of the legs. An MRI brain demonstrates herniation of the cerebellar
tonsils. Which of the following is not associated with Chiari type I malformation?
(a) Hydrocephalus
(b) Syringohydromelia
(c) Platyblasia
(d) Incomplete ossification of the C1 ring
(e) Myelomenigocoele

16
Ped Neuro
12 (e)
Chiari I is herniation of the cerebellar tonsils below a line connecting the basion and opisthion. Unlike Chiari II, Ill and
IV, Chiari I is not associated with myelomeningocoeles. Indeed, the condition is frequently isolated without
supratentorial anomalies. As well as the listed associations, Chiari I occurs with: basiliar impression, craniovertebral
fusion, and Klippel-Feil anomaly.
21 Which of the following is not a cause of secondary craniosynostosis?
(a) Crouzon's syndrome
(b) Hypothyroidism
(c) Previous shunt procedures
(d) Rickets
(e) Thalassaemia

21 (b)
Craniosynostosis is the premature closure of the sutures, which may be primary (idiopathic) or secondary. Secondary
causes include: metabolic (rickets, hypercalcaemia, hyperthyroidism, hypervitaminosis D), haematological
(thalassaemia, SCD), and bone dysplasias (achondroplasia, metaphyseal dysplasia). It is also associated with syndromes
(Crouzon, Apert, Treacher-Collins), and can occur following shunt surgery for hydrocephalus.
(CNS) 40 MR imaging demonstrates bilateral acoustic neuromas. Which of the following is not a recognised
association?
(a) Optic pathway gliomas
(b) Parasagittal meningomas
(c) -rarasplnal neurofibromas
(d) Facial nerve schwannomas
(e) Meningiomatosis

40 (a)
The patient fulfils the diagnostic criteria for Neurofibromatosis type 2. Unlike NF-1, there is no association with Usch
nodules, skeletal dysplasia, optic pathway gliomas or vascular dysplasia.
(CNS) 41 A neonate presents with a soft, painless mass in the posterior triangle of the neck. MR imaging demonstrates
a multiloculated, insinuating mass of intermediate signal on T1W and high signal on T2W. What is the most likely
diagnosis?
(a) Epidermoid cyst
(b) Dermoid cyst
(c) Lipoma
(d) Cystic hygroma
(e) Laryngocoele

41 (d)
The commonest 'soft' neck masses in infants and children are lipomas, haemangiomas and cystic hygromas. The latter
have low-tointermediate signal intensity on T1 W and high signal on T2W.

49 A 4 week old ex-premature baby has a witnessed seizure. A cranial US is performed which shows cystic structures
bilaterally, adjacent to the trigone of the lateral ventricles. Which of the following favours a diagnosis of chronic
periventricular leukomalacia over porencephaly?
(a) Anechoic cysts
(b) Persistenae of cysts on follow-up US
(c) Septated cysts
(d) Symmetrical distribution
(e) Watershed territory distribution

49 (d)
PVL is more common in preterm children, is secondary to ischaemia and usually occurs in the watershed areas. Initially
there will be hyperechoic changes which gradually become cystic (>2 wks); the cysts are never septated and usually
resolve over time. Porencephaly can be developmental or due to a vascular or infectious process which destroys brain
tissue; it is almost always asymmetrical, rarely disappears over time and is often seen as an extension of the ventricle or
sub-arachnoid space. If secondary to ischaemia it can also be in a watershed distribution.

17
Ped Neuro
(CNS) 51 MR imaging of a neonatal brain reveals an absent septum pellucidum, corpus callosum, third ventricle, and
interhemispheric fissure. The thalami are fused. Which variant of holoprosencephaly does this most likely represent?
(a) Alobar
(b) Bilobar
(c) Septo-optic dysplasia
(d) Lobar
(e) Semi lobar

51 (a)
This is the most profound form of holoprosencephaly, where there is almost no separation of the cerebral hemispheres
or ventricles. There is a 'horseshoe' configuration of neural tissue with a single crescentshaped ventricle. There is no
'bilobar' form of holoprosencephaly. Septo-optic dysplasia can be considered part of the holoprosencephaly spectrum.
53. Regarding rhabdomyosarcoma in the paediatric population. What is the most likely site of origin?
(a) Extremities
(b) Genito-urinary system
(c) Head and neck
(d) Orbits
(e) Retroperitoneal

53 (c)
Rhabdomyosarcoma represents 4-8% of cancers in children and is the 4th commonest after CNS tumours, neuroblastoma
and Willl)'s and is the commonest soft tissue sarcoma in children. The sites affected are: head and neck (28%),
extremities (24%), genitourinary system, trunk (11%), orbits (7%), and retroperitoneum (6%); other sites in <3%.
57 A 28 week antenatal US scan shows absence of the septum pellucidum and a radial array pattern of the medial
cerebral sulci. The ventricles are abnormal, with a dilated, elevated 3rd ventricle, disproportionate enlargement of the
occipital horns and small, widely separated frontal horns. Which of the following is not associated with this condition?
(a) Chiari II malformation
(b) Dandy-Walker syndrome
(c) Encephalocoele
(d) lnterhemispheric arachnoid cyst
(e) Lobar holoprosencephaly

57 (e)
The US features described are those of agenesis of the corpus callosum. Other associations include: alobar and semi-
lobar forms of holoprosencephaly, midline intracerebral lipoma, polymicrogyria, grey matter heterotopia and
porencephaly.
(MSK) 58 A 30 year old patient presents with multiple bilateral renal angiomyolipomas, one of which has bled. She is
also found to have a giant cell astrocytoma in her brain and bilateral interstitial lower lobe fibrosis on CXR. Which of
the following bone lesions is most commonly associated with this condition?
(a) Bone cysts
(b) Osteochondroma
(c) Giant cell tumour
(d) Fibrous dysplasia
(e) Adamantinoma

58 (a)
The underlying condition described is tuberous sclerosis. The associated bone cysts most commonly affect the small
bones of the hand. Other skeletal features include sclerotic bone islands which most commonly affect the calvarium (in
45% of cases) and also the pelvis and long bones.
59 A neonate has multiple apneic episodes, bradycardia, difficulty in swallowing, and increased tone in the upper limbs.
Cranial US shows hydrocephalus and absence of the corpus callosum. An MRI scan is performed which shows a small
posterior fossa. Which feature makes Chiari III rather than Chiari II malformation the likely diagnosis?
(a) Agenesis of the corpus callosum
(b) Encephalocoele
(c) Klippel-Feil anomaly
(d) Myelomeningocoele
(e) Syringohydromyelia

18
Ped Neuro
59 (b)
Chiari II malformation comprises a small posterior fossa, herniation of the tonsils and vermis through the foramen
magnum, myelomeningocoele (90%), obstructive hydrocephalus (90%), agenesis of the corpus callosum,
syringohydromyelia (50%) and abnormal cortical gyration. Chiari Ill is rare, has the features of Chiari II, but with an
associated encephalocoele. Chiari I is associated with Klippel-Feil anomaly.

60 A neonate is noted to have an enlarged head. Trans-cranial US confirms symmetrical dilation of the ventricles
consistent with hydrocephalus. Which of the following is true regarding congenital hydrocephalus?
(a) Aqueduct stenosis is the commonest cause
(b) It can be readily seen at the 12 week antenatal US
(c) It is more commonly associated with intra- rather than extracranial anomalies
(d) Most germinal matrix haemorrhage-related hydrocephalus requires shunting
(e) US is the most sensitive modality for identifying the cause

60 (a)
Aqueduct stenosis accounts for 43% of congenital hydrocephus. US assessment is difficult <20 wks because the
ventricles normally occupy a large percentage of the cranial vault. The most sensitive modality for determining the cause
(not presence) of hydrocephalus is MRI. Grade III germinal matrix haemorrhage has hydrocephalus, but in 2/3 this is
stable and requires no treatment. Intracranial anomalies are associated in 37% (e.g. agenesis corpus callosum),
extracranial anomalies in 63% (e.g. VSD, spina bifida).

(CNS) 67 A patient presents with a neurocutaneous disorder. Which of the following is not a recognised association?
(a) Neurofibromatosis-1 and plexiform neurofibromas
(b) Osler-Weber-Rendu syndrome and cavernomas
(c) Von Hippel Lindau disease and pheochromocytomas
(d) Tuberous sclerosis and pial enhancement
(e) Sturge-Weber syndrome and cerebral atrophy

67 (d)
Tuberosis sclerosis is associated with, amongst other features: cortical tubers, subependymal nodules, giant cell
astrocytomas, white matter lesions, retinal phakomas and vascular abnormalities.

(CNS) 69 A child undergoes MR imaging of their spine. A review of the lower lumbar levels reveals that the dorsal dura
is incomplete and the subarachnoid space lies ventral to a neural placode, which is directly continuous with the skin.
The ventral subarachnoid space does not appear to be dilated. What would be the most appropriate term to describe this
spinal dysraphism?
(a) Myelocoele
(b) Myelomeningocoele
(c) Lipomyelocoele
(d) Lipomyelomeningocoele
(e) Spinal lipoma

69 (a)
In the case above, if the ventral subarachnoid space were dilated, this would be more appropriately termed a
myelomeningocoele. If the dorsal placode were continuous with a lipoma, continuous with the subcutaneous tissues,
this would represent a lipomyelocoele.

(CNS) 71 A child presents with a lump in the neck, and a branchial cleft cyst is suspected. Which of the following
features would be unusual for this diagnosis?
(a) Attenuation of 10 HU on unenhanced CT
(b) High signal on T2W MR imaging
(c) Located deep to a sternomastoid muscle
(d) Lack of rim enhancement on CT
(e) Present in the posterior triangle of the neck

71 (e)
Branchial cleft cysts are derived from the 1st or (more commonly) the 2nd cervical pouch. These are usually anterior
triangle lesions and of water CT attenuation (< 20 HU). Rim enhancement is seen in infected cysts.

19
Ped Neuro
(CNS) 72 A 42 year old woman presents with weakness in her hands. On examination she is noted to have wasting of
the small muscles of the hand, burns and cuts to her fingers. There are reduced biceps reflexes, increased tone in the
legs and upgoing plantars. An MRI brain and spine are requested. What is the 1most likely unifying diagnosis?
(a) Cervical spondylosis
(b) Chiari II malformation
(c) Motor neurone disease
(d) Multiple sclerosis
(e) Rheumatoid arthritis

72 (b)
The patient has lower motor neurone signs in the upper limbs and upper motor neurone signs in the lower limbs. The
most likely diagnosis is a syrinx (syringohydromyelia) within the cervical-cord. A Chiari II malformation is usually
accompanied by a myelomeningocele, abnormal development of the cerebellar vermis and hydrocephalus. The herniated
cerebellum blocks CSF circulation and leads to the formation of a syrinx within the Spinal cord.
Currie
(CNS) 8.Routine first-trimester antenatal ultrasound scan reveals a large posterior fossa cyst and ventriculomegaly. Fetal
MRI demonstrates dysgenesis of the corpus callosum, a large posterior fossa and hypoplasia of the cerebellar vermis.
What is the most likely diagnosis?
a. Dandy-Walker malformation
b. Dandy-Walker variant
c. Megacisterna magna
d. Arachnoid cyst
e. Porencephaly

8.a. Dandy-Walker malformation


Dandy-Walker malformation is characterised by an enlarged posterior fossa (not seen in Dandy-Walker variant) with
high-rising tentorium cerebelli, dys/agenesis of the cerebellar vermis (intact in megacisterna magna) and cystic dilatation
of the fourth ventricle (normal in arachnoid cyst). Ventriculomegaly is also common.
16. The mother of a three week old child notices a mass in her baby’s lower neck. The child is otherwise well. There is
a history of normal pregnancy and the child was delivered by forceps. Ultrasound scan reveals homogeneous
enlargement of the lower third of the right sternocleidomastoid muscle but no focal lesion is identified. T2-weighted
MRI shows diffuse abnormal high signal intensity over the same area. The most likely diagnosis is:
a. Haematoma
b. Branchial cleft cyst
c. Fibromatosis colli
d. Neuroblastoma
e. Cystic hygroma

16. c. Fibromatosis colli


This is a rare form of infantile fibromatosis that occurs solely within the sternocleidomastoid muscle. In the vast majority
it is associated with birth trauma (e.g. forceps delivery). This is thought to lead to compartment syndrome, pressure
necrosis and secondary fibrosis of the muscle. It usually locates to the lower third of the muscle, between the sternal and
clavicular heads, and is usually unilateral.
Ultrasound may reveal a well- or ill-defined mass or may just show homogeneous muscle enlargement. In approximately
two-thirds of individuals, the abnormality spontaneously regresses by the age of two. Expected ultrasonographic
appearances of a haematoma include a heterogeneous mass of mixed cystic and solid components.
(CNS) 18. A 28 year old woman presents with a mass in her neck. She gives a history of multiple parotid abscesses
which have been refractory to drainage and antibiotics. The mass is located at the anteromedial border of her right
sternocleidomastoid muscle. Ultrasound demonstrates a compressible mass with internal debris which is devoid of
internal flow on Doppler imaging. MRI shows a cystic mass consisting of a curved rim of tissue pointing medially
between the internal and external carotid arteries. There is slight capsular enhancement. What is the most likely
diagnosis?
a. Cervical abscess
b. Necrotic neural tumour
c. Submandibular gland cyst
d. Necrotic inflammatory lymphadenopathy
e. Second branchial cleft cyst

20
Ped Neuro
18. e. Second branchial cleft cyst
Failure of involution of branchial clefts can lead to branchial cleft cysts, fistulae and/or sinuses. Second branchial cleft
cysts account for 95% of all branchial cleft anomalies. Male and female incidence is equal and the typical age of
presentation is 10–40 years. Second branchial cleft cysts are classified into four types depending on their location. The
most common is type II, which occurs along the anterior surface of the sternocleidomastoid muscle, lateral to the carotid
space and posterior to the submandibular gland adhering to the great vessels. On CT/MR the ‘beak sign’ is
pathognomonic. This is a curved rim of tissue pointing medially between the internal and external carotid arteries.

29. A two month old child is brought to hospital as his parents have noticed he has become more floppy over the
preceding weeks. Examination reveals marked hypotonia, head lag and increased head circumference (>98th percentile).
CT brain shows low-density white matter. T2-weighted MRI demonstrates diffuse, symmetric increased signal intensity
throughout the white matter. There is relative sparing of the internal and external capsules and also the corpus callosum.
Both globus pallidi show high signal intensity but there is relative sparing of the putamen and caudate nucleus. Which
of the following would fail to confirm the diagnosis?
a. Brain biopsy
b. Proton MR spectroscopy
c. Quantitative urine study
d. Fibroblast cultures
e. Diffusion-weighted MRI

29. e. Diffusion-weighted MRI


The history and MRI findings are suggestive of Canavan disease. This is an autosomal recessive disorder due to
deficiency of the enzyme aspartoacylase. Accumulation of N-acetylaspartic acid (NAA) in the brain pursues and leads
to leukodystrophy. Histology reveals soft and gelatinous white matter. Change is most prominent in the deeper cortex
and subcortical white matter, with relative sparing of the deeper white matter and internal capsule. On T2-weighted
MRI, the globus pallidus is always of high signal intensity, with frequent involvement of the thalamus but with relative
sparing of the caudate and putamen. The main differential diagnosis on imaging is Alexander disease, a rare disorder
which shows no pattern of inheritance. Both conditions are leukodystrophies with macrocrania. Brain biopsy can be
used to differentiate. Diagnosis of Canavan disease is also possible with proton MR spectroscopy which shows a
characteristic increase in the NAA peak. Levels of NAA are also abnormally high in urine and there will be a deficiency
of aspartoacyclase in cultured skin fibroblasts. Aspartoacyclase is not present in plasma or blood cells. Diffusion-
weighted MRI will add little to the information already obtained with T2-weighted imaging.
(CNS) 32. Which of the following neurological imaging findings would offer a clear differential diagnosis between
tuberous sclerosis and neurofibromatosis Type I?
a. Basal ganglia calcification
b. Optic nerve glioma
c. Cerebral astrocytoma
d. Enhancing heterotopic grey matter
e. Multiple foci of hyperintensity on T2-weighted imaging

32. d. Enhancing heterotopic grey matter


All of the other features may be found in either condition. Giant cell astrocytomas located in the region of the foramen
of Monro in tuberous sclerosis may degenerate into high-grade astrocytomas. Cortical tubers show as multiple nodules
which are hyperintense on T2/FLAIR imaging. CNS hamartomas (occurring in up to 75-90% of NF1) also display these
characteristics - they are often termed ‘unidentified bright objects’.

(CNS) 43. An eight year old boy with skin hyperpigmentation presents with deteriorating vision and hearing loss. CT
shows large symmetric low-density lesions in the parietaloccipital white matter. T2-weighted MR imaging demonstrates
confluent symmetric hyperintensity within the parietal-occipital white matter extending across the splenium of the
corpus callosum. There is relative sparing of the frontal lobes. Follow-up MRI six months later shows cerebral atrophy
and more extensive white matter change with involvement of the frontal lobes and cerebellum. At this point the patient
had developed spastic quadriplegia. What is the most likely diagnosis?
a. Multiple sclerosis
b. Lymphomatoid granulomatosis
c. Acute disseminated encephalomyelitis
d. Adrenoleukodystrophy
e. Autoimmune vasculitis

21
Ped Neuro
43. d. Adrenoleukodystrophy (ALD)
ALD is an inherited metabolic disorder characterised by progressive demyelination of cerebral white matter. It is
commonly X-linked recessive and boys present between the ages of 3 and 12 years with ataxia, deteriorating vision,
hearing loss, altered behaviour and mental deterioration. It is associated with adrenal insufficiency (Addison’s disease).
Predominantly, there is posterior white matter involvement with the disease advancing toward the frontal lobes and
cerebellum. Imaging shows CT hypodensity, MR T1 hypointensity and T2 hyperintensity. Administration of contrast
shows enhancement of the lateral margins of the lesions, indicating areas of active demyelination.
Acute disseminated encephalomyelitis is an autoimmune reaction against the patient’s white matter. It may present
within days or weeks following an exanthematous viral reaction or vaccination. Imaging demonstrates multifocal
hypodense (CT)/hyperintense (T2 MRI), usually asymmetrical, white matter abnormalities. Corticosteroids result in
dramatic improvement and follow-up scans show no additional lesions.

(CNS) 48. A five year old boy presents with visual fixation. The ophthalmologist suspects optic nerve hypoplasia. Brain
CT confirms optic nerve hypoplasia, an absent septum pellucidum and a third cerebral abnormality. What is the third
abnormality likely to be?
a. Schizencephaly
b. Cortical dysplasia
c. Polymicrogyria
d. Agenesis of the corpus callosum
e. Type II Chiari malformation

48. a. Schizencephaly
The combination of absent septum pellucidum and optic nerve hypoplasia is indicative of septooptic dysplasia (SOD).
Most patients present in infancy with visual disturbance, seizures or endocrine abnormalities (pituitary dysfunction is
seen in approximately 50% of cases). Additional abnormalities are often present with schizencephaly being the most
common (50%).

(CNS) 51. A neurologist requests an MRI scan of a patient with longstanding temporal lobe epilepsy. He suspects mesial
temporal sclerosis. What imaging plane would you pay particular attention to and what would you expect to find in this
condition?
a. Axial plane demonstrating volume loss and reduced signal intensity of the parahippocampal gyrus on T2-weighted
imaging
b. Coronal plane demonstrating volume loss and increased signal intensity of the hippocampus on T1-weighted imaging
c. Coronal plane demonstrating volume loss and increased signal intensity of the parahippocampal gyrus on T2-weighted
imaging
d. Axial plane demonstrating volume loss and reduced signal intensity of the hippocampus on T1-weighted imaging
e. Coronal plane demonstrating volume loss and increased signal intensity of the hippocampus on T2-weighted imaging

51. e. Coronal plane demonstrating volume loss and increased signal intensity of the hippocampus on T2-weighted
imaging
Mesial temporal sclerosis (hippocampal sclerosis) is the most common lesion associated with temporal lobe epilepsy.
Acquisition in the coronal plane is mandatory for its detection. MRI typically shows volume loss and increased signal
on T2-weighted imaging.

(CNS) 54. A three year old girl undergoes further investigation for refractory seizures. Contrast-enhanced T1-weighted
imaging shows diffuse pial enhancement of variable thickness over the parieto-occipital region of the right cerebral
hemisphere. There is atrophy of the underlying cerebrum and the right choroid plexus is enlarged. Several hypointense
foci are seen within the gyri and adjacent white matter. There is also bilateral well-defined orbital choroidal
enhancement. T2-weighted imaging demonstrates prominent superficial cortical veins. What is the most likely
diagnosis?
a. Klippel-Trenaunay syndrome
b. Sturge-Weber syndrome
c. Wyburn-Mason syndrome
d. Neurofibromatosis
e. Tuberous sclerosis

22
Ped Neuro
54. b. Sturge-Weber syndrome
Sturge-Weber syndrome is a congenital disease characterised by capillary venous angiomas involving the face (port-
wine stain, usually ophthalmic division of trigeminal nerve), choroid of the eye and leptomeninges. Clinical
manifestations include focal seizures (80% in the first year of life), developmental delay, hemiparesis and homonymous
hemianopia. Seizures typically become refractory to medication.
Leptomeningeal angiomas are confined to the pia mater and occur primarily within the parieto-occipital region. There
is cortical hemiatrophy beneath the angioma due to local anoxia and usually after the age of two years there is cortical
calcification manifesting as low signal intensity on T1 post-contrast images. Other findings include enlargement of the
ipsilateral choroid plexus, dilatation of the transparenchymal veins that communicate between the superficial and deep
cerebral venous systems, and orbital choroidal haemangiomas.

Grainger
(CNS) QUESTION 5
A 12-year-old child is suspected to have a diagnosis of neurofibromatosis. Which one of the following radiological
findings would favour a diagnosis of neurofibromatosis type 1 over neurofibromatosis type 2?
A Bilateral acoustic neuromas
B Leptomeningeal angiomas
C Multiple meningiomas
D Sphenoid wing hypoplasia
E Spinal ependymoma

ANSWER: D.
QUESTION 6
A mother is concerned about the shape of her 2-year-old son's head. The GP agrees that it appears elongated and he is
referred for skull radiographs and a CT head to look for evidence of craniosynostosis. Which one of the following
statements is true regarding congenital skull abnormalities?
A Apert’s syndrome is associated with sagittal synostosis.
B Brachycephaly is associated with a higher incidence of neurological abnormalities compared with scaphocephaly.
C Crouzon syndrome affects only the coronal sutures.
D Sagittal synostosis is often seen with hydrocephalus.
E Synostosis of the lamboid suture is more common than the sagittal

ANSWER: B
The sagittal suture is the most commonly affected in primary craniosynostosis. Involvement of the coronal suture (in
brachycephaly) is often associated with clinical syndromes.

(CNS) QUESTION 10
A 22-month-old child with developmental delay presents with seizures. The MRI findings include hydrocephalus with
a markedly dilated fourth ventricle and hypoplasia of the cerebellar vermis. Which one of the following is the most
likely diagnosis?
A Chiari II malformation
B Dandy-Walker malformation
C Encephalocele
D Holoprosencephalv
E Joubert's syndrome

ANSWER: B
The Dandy-Walker malformation describes variable hypoplasia of the cerebellar vermis and a large cystic collection
which is continuous with the fourth ventricle in the posterior fossa.

23
Ped Neuro
(CNS) QUESTION 25
A 7-year-old boy is referred from the paediatric clinic for an MRI brain to investigate learning difficulties and abnormal
gait. The MRI reveals high T2 signal in the splenium and posterior body of the corpus callosum as well as in the
peritrigonal white matter. There is marginal enhancement at the anterior edge of the abnormal area. Which one of the
following is the most likely diagnosis?
A Adrenoleukodystrophy
B Alexander disease
C Kearns-Sayer syndrome
D Krabbe’s leukodystrophy
E Maple syrup urine disease

ANSWER: A
This is the most common leukodystrophy of children. Demyelination begins in the posterior central white matter and
progresses to the corticospinal tracts and visual and auditory pathways. The leading edge shows enhancement due to
active inflammation.

QUESTION 25
An infant born at 38 weeks' gestation has suffered hypoxic birth trauma. In which of the following locations are you
most likely to see abnormalities on MRI?
A Cerebellar peduncles
B Midbrain
C Periventricular region
D Subcortical white matter of the frontal cortex
E Thalami

ANSWER: E
If an infant suffers hypoxic damage at term, the areas most affected are those that are most metabolicaliy active. These
include the putamen, thalami and adjacent white matter.

(CNS) QUESTION 27
A 3-year-old boy has an MR1 brain which shows descent of the cerebellar tonsils below the foramen magnum. Which
one of the following is more likely to suggest a Chiari I rather than a Chiari II malformation?
A Craniosynostosis
B Elongation of the medulla
C Hydrocephalus
D Myelomeningocoele
E Tectal beaking

ANSWER: A
Chiari I may be acquired under conditions with raised intracranial pressure, decreased intraspinal pressure or decreased
posterior fossa volume. Hydrocephalus and elongation of the medulla may be features but these are also seen in Chiari
II. Myelomeningocoele and tectal beaking are features of Chiari II.
(GU) QUESTION 30
Which one of the following radiological findings is a recognised feature of Von Hippel Lindau (VHL) disease?
A Bilateral adrenal masses that yield a high signal on T2w sequences
B Cerebral aneurysms on CT angiography
C Evidence of calcified subependymal nodules on CT head
D Polymicrogyria and corpus callosum agenesis on MRI brain
E Unenhanced CT head demonstrating a midline, hyperdense vermian mass abutting the roof of the 4th ventricle

ANSWER: A
Phaeochromocytomas are hyperintense on T2w sequences and iso- or hypointense to the liver on Tlw sequences.

24
Ped Neuro
QUESTION 38
A large posterior fossa cyst is identified during an antenatal ultrasound scan. Following delivery the diagnosis of a
Dandy Walker malformation is being considered. In addition to a large posterior fossa cystic mass which one of the
following abnormalities would support this diagnosis?
A Agenesis of the septum pellucidum
B Hypoplastic cerebellar vermis
C Interiorly displaced fourth ventricle
D Myelomeningocoele
E Tectal plate beaking

ANSWER: B
The Dandy Walker malformation is a cystic dilatation of the fourth ventricle. The main differential is a posterior fossa
arachnoid cyst; however, this does not communicate with the fourth ventricle.
(CNS) QUESTION 38
A 6-year-old girl with several pigmented patches on her skin presents with deteriorating vision. An MRI brain shows
fusiform expansion of both optic nerves, with enlargement of the optic foramina. In addition there are multiple small
lesions in the basal ganglia and pons which are hyperintense on T2w images. What is the most likely diagnosis?
A Neurofibromatosis type 1 with optic nerve gliomas and cerebral hamartomas
B Neurofibromatosis type 2 with optic nerve gliomas and cerebral hamartomas
C Neurofibromatosis type 2 with optic nerve meningiomas and multiple schwannomas
D Tuberous sclerosis with cortical tubers and retinal hamartomas
E Tuberous sclerosis with subependymal and retinal hamartomas

ANSWER: A.
(CNS) QUESTION 49
An 18-month-old child with a facial port wine stain presents with reduced movement on the left side and developmental
delay. Her mother also reports left-sided focal seizures. What are the most likely radiological findings?
A Atrophy of the left cerebral hemisphere with enhancement overlying the left parietal cortex and enlargement of the
right choroid plexus
B Atrophy of the right cerebral hemisphere with enhancement overlying the right parietal cortex and enlargement of the
right choroid plexus
C Cystic dilatation of the fourth ventricle with hypoplasia of the vermis and hydrocephalus
D Hydrocephalus, inferior displacement of the cerebellar tonsils and elongation of the fourth ventricle
E Multiple small calcified subependymal nodules, a partly cystic mass at the foramen of Monro and several retinal
lesions

ANSWER: A
The clinical history describes Sturge-Weber syndrome. The imaging findings include leptomeningeal angiomas on the
same side as the facial port wine.

(GIT) QUESTION 49
A 27-year-old woman is referred to the gastroenterology outpatient clinic with a 3-month history of upper abdominal
pain. There is no past medical history of note, but her sister has recently been diagnosed with a ‘brain tumour’.A contrast-
enhanced CT (portal venous phase) of the abdomen demonstrates amulticystic lesion in the head of the pancreas. The
lesion contains 10 cysts measuring up to 15 mm in size with a small amount of calcification centrally. Several larger
cysts are present in the pancreatic body and tail and both kidneys contain cortical cysts. What is the most likely
underlying diagnosis?
A Autosomal dominant polycystic kidney disease
B Cystic fibrosis
C HHT
D Tuberous sclerosis
E Von Hippel Lindau disease (VHL)

ANSWER: E
The cystic pancreatic lesion is likely to be a serous cystadenoma and, in combination with simple pancreatic and renal
cysts, is consistent with VHL.

25
Ped Neuro
(CNS) QUESTION 53
A 21-year-old man has an ultrasound of the abdomen which shows bilateral renal masses and several pancreatic cysts.
A diagnosis of von Hippel Lindau disease is being considered. The presence of which of the following intracranial
tumours would be strongly supportive of this diagnosis?
A Astrocytoma
B Craniopharyngioma
C Haemangioblastoma
D Medulloblastoma
E Optic nerve glioma

ANSWER: C

QUESTION 61
Primary sagittal synostosis is suspected in a 3-month-old infant and a series of plain skull radiographs are taken. Which
one of the following statements is
A Lambdoid and sagittal sutures are examined on the Townes' projection.
B The AP projection is the best view to assess the foramen magnum and the fontanelles.
C The AP projection will only assess lambdoid and metopic sutures.
D The AP projection will only assess the coronal and the sagittal sutures.
E The sagittal and lambdoid sutures are examined on a lateral projection.

ANSWER: A
Standard radiographs for craniosynostosis assessment includes Townes’, AP and lateral projections.

QUESTION 70
A 2-year-old infant presents with a history of developmental delay, seizures and subcutaneous lesions. The clinicians
suspect tuberous sclerosis. Which one of the following radiological findings are consistent with this diagnosis?
A Calcified subependymal nodules
B Leptomeningeal angiomas
C Multiple meningiomas
D Pilocytic astrocytoma
E Retinoblastoma

ANSWER: A
Subependymal nodules are the most common neurological abnormality, seen in 85—95% of patients.

QUESTION 79
A 10-month-old girl is being investigated for delayed motor development. An MRI brain is performed under sedation.
Which one of the following statements is correct regarding the MRI appearances of normal brain development in a child
of this age?
A Adult appearances of the corpus callosum are not expected.
B It is possible to assess myelination accurately using Tlw images alone.
C On Tlw images, myelination will have almost reached adult maturity by imaging criteria
D On T2w images, myelination of the optic radiation is abnormal.
E On T2w images, subcortical white matter myelination is seen extending from the frontal cortex into the parietal and
temporal lobes.

ANSWER: C
During the first 6 months of life, brain maturation is easiest to follow on Tlw images, where the appearance of myelin
is bright. By 10 months the Tlw images will have an almost adult appearance, and further changes are easier to assess
using the T2w images.

26
Ped Neuro
(CNS) QUESTION 80
An 18-morith-old boy with multiple skin lesions, developmental delay and seizures has an MRI of the brain. This
demonstrates several small subependymal nodules that are isointense to white matter on T2w images and project into
the lateral ventricles. There is also a small, well-demarcated mass at the foramen of Monro which is hyperintense on
T2w images and displays uniform enhancement. What is the most likely underlying diagnosis?
A Neurocutaneous melanosis
B Medulloblastoma
C Neurofibromatosis type 1
D Neurofibromatosis type 2
E Tuberous sclerosis

ANSWER: E
This patient has subependymal hamartomas (which can calcify) and a giant cell astrocytoma. Other CNS features include
cortical tubers.
Oxford
2. A baby boy is born prematurely at 30 weeks gestation. Cranial ultrasound demonstrates bilateral multiseptate
cystic lesions within the frontal lobe white matter with associated ex vacuo dilatation of the ventricles. Which of
the following is the most likely diagnosis?
A. Periventricular leucomalacia.
B. Porencephaly.
C. Supratentorial arachnoid cysts.
D. Vein of Galen malformation.
E. Subependymal cysts.

2. A. Periventricular leucomalacia.
This refers to white matter necrosis, typically involving the centrum semiovale, and is seen in premature infants. This
results from hypoxic-ischaemic injury at the watershed areas, which in premature infants are present in a periventricular
location. Porencephaly refers to an area of encephalomalacia, which may or may not communicate with the ventricular
system and develops postnatally or in the third trimester. This is the end result of a destructive process, such as an
intraparenchymal haemorrhage. Sylvian fissure cysts are the most common site for supratentorial arachnoid cysts. Vein
of Galen malformations occur in the midline and exhibit Doppler flow. Subependymal cysts are detected in the
caudothalamic groove.
7. A 2-year-old girl is investigated for slow motor development via MRI. Which of the following radiological
features would suggest a diagnosis of Dandy- Walker malformation, as opposed to Dandy-Walker variant?
A. Cerebellar dysgenesis.
B. Enlargement of the posterior fossa.
C. Agenesis of the corpus callosum.
D. Holoprosencephaly.
E. Cystic dilatation of the fourth ventricle.

7. B. Enlargement of the posterior fossa.


Dandy-Walker variant is more common, accounting for a third of all posterior fossa malformations, but less severe than
the malformation. Enlargement of the posterior fossa is not a feature. Cystic dilatation of the fourth ventricle with
vermian dysgenesis is characteristic of both. Associated CNS anomalies, usually of the midline, are also seen, as is
ventriculomegaly, although both are more common with Dandy-Walker malformation.
(CNS) 7. A patient with a known diagnosis of a neurocutaneous syndrome is having a routine follow-up MRI
scan. This patient is noted to have a history of right retinal calcifications. On MRI a retinal lesion is noted in the
right eye, which is increased signal on T1WI. The scan shows a 2-cm lesion in the left lateral cerebellar
hemisphere. This is predominantly low on T1WI and high on T2WI, but has a peripheral nodular area, which
has a central signal void and demonstrates enhancement on T1WI. A further small enhancing lesion is noted in
the cervical spinal cord, with an associated syrinx. The appearances are unchanged from previous imaging. Based
on these findings, which neurocutaneous syndrome does this patient have?
A. Neurofibromatosis type 1 (NF-1).
B. Neurofibromatosis type 2 (NF-2).
C. Tuberous sclerosis.
D. Sturge Weber syndrome.
E. Von Hippel-Lindau syndrome.

27
Ped Neuro
7. E. Von Hippel-Lindau syndrome.
This syndrome is characterized by retinal lesions variously described as being retinal angiomas, retinal haemangiomas,
or retinal hamartomas. These lesions (Lindau tumours) cause retinal calcification, although this is also seen in NF-1 and
tuberous sclerosis (TS) from other causes. The findings describe the classical appearance of a cerebellar
haemangioblastoma. These lesions can also be entirely cystic or solid in a minority of cases. A signal void and lack of
dural enhancement help differentiate this from other infratentorial masses in adults. Twenty per cent of
haemangioblastomas are associated with Von Hippel-Lindau syndrome and 80% of Von Hippel-Lindau syndrome
patients have this tumour. The spinal finding also indicates a further haemangioblastoma in the cord. Unlike in the
posterior fossa, these seldom have flow voids, but are associated with the development of a syrinx.
21. A 12-year-old boy is investigated via MRI brain for headache, nystagmus, and ataxia. Which of the following
radiological findings would suggest a diagnosis of Chiari I malformation as opposed to Chiari II?
A. Lacunar skull.
B. Myelomeningocoele.
C. Elongation of the fourth ventricle.
D. Caudal displacement of the cerebellar tonsils.
E. Cervicomedullary kinking.

21. D. Caudal displacement of the cerebellar tonsils.


Chiari II is seen in all patients with open spinal dysraphisms, such myelomeningocoele. Lacunar skull (luckenshadel) is
also associated with Chiari II. Cervicomedullary kinking is common to both, although more so with Chiari II. Caudal
displacement of the cerebellar tonsils is a feature of Chiari I, whereas in Chiari II the vermis herniates into the foramen
magnum and the tonsils are lateral to the medulla.
26. A 4-year-old boy is investigated via MRI brain for developmental delay and intractable seizures. Which of
the following findings is in keeping with a diagnosis of schizencephaly?
A. Intracerebral cleft lined by gray matter connecting the lateral ventricle to the subarachnoid space.
B. Smooth cortical surface with absence of convolutions.
C. Multiple small, irregular cortical convolutions without intervening sulci.
D. Column of grey matter extending from the subependymal to the pial surface.
E. Circumferential, symmetric band of heterotopic grey matter deep to the cortical surface.

26. A. Intracerebral cleft lined by gray matter connecting the lateral ventricle to the subarachnoid space.
Schizencephaly can be defined as open or closed lip, depending on the presence of separation of the cleft walls. The
remaining options describe lissencephaly, polymicrogyria, transmantle heterotopia, and subcortical band heterotopia,
respectively. Transmantle heterotopia can potentially be confused with closed lip schizencephaly.
31. A 14-year-old boy is having a follow-up MRI brain for a known seizure disorder. Axial T2WI demonstrates
gyriform low signal in the left occipital and temporal lobes with corresponding volume loss. Leptomeningeal
enhancement is present on the axial T1WI post contrast. A right-sided developmental venous anomaly (DVA) is
also present. What is the most likely diagnosis?
A. Neurofibromatosis type 1.
B. Neurofibromatosis type 2.
C. Sturge Weber syndrome.
D. Tuberous sclerosis.
E. Von Hippel-Lindau disease.

31. C. Sturge Weber syndrome.


The gyriform low T2WI signal corresponds to cortical calcification, which in association with unilateral atrophy and
leptomeningeal enhancement is characteristic of Sturge Weber syndrome, one of the neurocutaneous phakomatoses.
Other findings include calvarial thickening and choroid plexus angiomas. Associated facial port wine stain in the
distribution of the trigeminal nerve is classic.
36. A 12-year-old boy presents with a painless neck mass which recently increased in size after an upper
respiratory tract infection. Which of the following radiological findings are in keeping with a second branchial
cleft cyst?
A. Anechoic cystic mass posterior to sternocleidomastoid in the posterior triangle.
B. Lateral echogenic mass with hypoechoic vascular channels.
C. Anechoic cystic mass anterior to sternocleidomastoid near the angle of the mandible.
D. Anechoic cystic mass in an infrahyoid midline location.
E. Anechoic cystic mass inferior and posterior to the tragus.

28
Ped Neuro
36. C. Anechoic cystic mass anterior to sternocleidomastoid near the angle of the mandible.
The other options are typical locations for cystic hygroma, infantile haemangioma, thyroglossal cyst, and first branchial
cleft cyst, respectively. The majority (75%) of branchial cleft cysts are remnants of the second branchial cleft. Cystic
hygromas are lymphatic malformations (lymphangiomas) that result from blockage of lymphatic channels. Most present
before 2 years of age. Most are slow growing, but can suddenly enlarge following infection or haemorrhage into the
lesion. Infantile haemangiomas usually grow rapidly until 9-10 months of age, followed by spontaneous resolution,
which can take up to 10 years. Thyroglossal cysts are remnants of the thyroglossal duct, which extends from the foramen
caecum at the base of the tongue to the pyramidal lobe of the thyroid. The majority (65%) are infrahyoid.
(GIT) A 35-year-old female undergoes an MRI of abdomen that shows multiple cystic lesions in the pancreas.
Each lesion consists of a cluster of small cysts with central scar. Multiple cysts and solid lesions are also noted in
both kidneys. What further investigation/s would you recommend?
A. Ophthalmology referral.
B. MRI of the brain.
C. MRI of the spine.
D. Molecular genetic testing and genetic counselling.
E. All of the above.

E. All of the above.


All the findings are manifestations of von Hippel–Lindau (VHL) disease. VHL is a rare, inherited, multisystem disorder
characterized by the development of multiple benign and malignant neoplasms. It is an autosomal dominant disorder
caused by inactivation of a tumour suppressor gene located on chromosome 3p25.5. The clinical manifestations are
broad and include central nervous system (CNS) and retinal haemangioblastomas, renal cysts and tumours, pancreatic
cysts and tumours, phaechromocytomas, endolymphatic sac tumours, and epididymal cystadenomas. The diagnostic
criteria for VHL include:
(i) >1 CNS haemangioblastoma,
(ii) one CNS haemangioblastoma + visceral manifestations of VHL
(iii) any manifestation + known family history of VHL.
The most common causes of death in VHL are renal cell carcinoma and neurologic complications of cerebellar
haemangioblastomas. Genetic counselling & screening are important in early detection & Rx of VHL lesions
41. An 18-month-old girl presents with increasing incoordination and developmental regression. T2WI
demonstrates confluent high signal within the periventricular white matter and centrum semiovale, with
radiating linear low signal intensity, giving a ‘tigroid’ pattern. Sparing of subcortical U fibres is also noted. What
is the most likely diagnosis?
A. Krabbe disease.
B. Metachromatic leucodystrophy.
C. X-linked adrenoleucodystrophy.
D. Alexander disease.
E. Canavan disease.

41. B. Metachromatic leucodystrophy.


Leucodystrophies are dysmyelinating inherited white matter diseases, which are secondary to lysosomal, peroxisomal,
or mitochondrial dysfunction. Metachromatic leucodystrophy is caused by deficiency of the lysosomal enzyme
ayrlsulfatase A. The ‘tigroid’ pattern relates to sparing of perivascular white matter.
Table 5.1 outlines characteristic radiological findings of the major leucodystrophies.
Table 5.1 Characteristic radiological findings of the major leucodystrophies.
Organelle
Disease
dysfunction Radiological findings

Metachromatic U fibre sparing


leucodystrophy Perivascular sparing (tigroid pattern)
Lysosome
Krabbe disease Thalamic and caudate nuclei high attenuation on CT U fibre sparing
Mucopolysaccharidosis Well-defined high T2 signal foci in corpus callosum and basal ganglia
X-linked Symmetrical peritrigonal and splenium high T2 signal
Peroxisome
adrenoleucodystrophy Peripheral contrast enhancement
Mitochondria MELAS Multiple cortical and subcortical infarct-like lesions
Canavan disease U fibers preferentially affected
Unclassified
Alexander disease Frontal lobe predilection Subcortical white matter affected early

29
Ped Neuro
(CNS) 43. A 4-week-old infant with a history of breech delivery is brought to the A&E department with a history
of swelling in the left side of the neck and torticollis. An ultrasound of the neck demonstrates a non-tender, focal
fusiform enlargement of the lower half of the left sternocleidomastoid muscle. No other abnormality is identified.
What is the diagnosis?
A. Fibromatosis colli.
B. Lymphoma.
C. Rhabdomyosarcoma.
D. Cystic hygroma.
E. Branchial cleft cyst.

43. A. Fibromatosis colli.


Fibromatosis colli or pseudotumour of the sternocleidomastoid muscle is a benign self-limiting condition that occurs in
the first 2–4 weeks of life. It is often associated with breech or forceps delivery. It is thought to result from pressure
necrosis and subsequent fibrocollagenous infiltration of the sternocleidomastoid muscle. Typical presentation is with a
firm and non-tender mass in the lower two-thirds of the sternocleidomastoid muscle. It may be associated with torticollis
in approximately 20% of cases. It usually resolves spontaneously in the first year of life. Typical findings on ultrasound include
focal or diffuse enlargement of the sternocleidomastoid with variable echogenicity. The clinical history and ultrasound appearances
are so typical that no further investigation is usually necessary.

47. A premature baby girl is noted to have a skull deformity consistent with scaphocephaly. Fusion of which
vault suture or sutures gives rise to this craniosynostosis?
A. Coronal suture.
B. Sagittal suture.
C. Lambdoid suture.
D. Metopic suture.
E. Sagittal, coronal, and lambdoid sutures.

47. B. Sagittal suture.


Craniosynostosis refers to premature closing of sutures and is often present at birth. It may be primary or secondary to
bone dysplasias or haemoglobinopathy, or as part of a generalized syndrome. Scaphocephaly is the most common
craniosynostosis and results in a long skull.
Brachycephaly arises from bilateral closure of the coronal suture, resulting in a short, tall skull. Unilateral fusion of
the lambdoid suture is seen in plagiocephaly, giving a lopsided skull.
Trigonocephaly is a forward-pointing skull from premature closure of the metopic suture.
Intrauterine closure of coronal, sagittal and lambdoid sutures gives rise to the cloverleaf skull, which may be
associated with thanatophoric dysplasia.

(CNS) 54. A 30-year-old male presents with a fluctuant swelling in the right side of the neck. On ultrasound
examination, an anechoic lesion with posterior acoustic enhancement is noted along the anteromedial margin of
the sternocleiodomastoid muscle, posterior to the submandibular gland and superficial to the carotid artery and
internal jugular vein. There is no increased surrounding vascularity on power Doppler. What is the likely
diagnosis?
A. Dermoid cyst.
B. Lymphangioma.
C. First branchial cleft cyst.
D. Abscess.
E. Second branchial cyst.

54. E. Second branchial cleft cyst.


The majority of branchial cleft anomalies arise from the second branchial cleft. The described findings are typical of a
second branchial cleft cyst. Similar ultrasound appearances may be seen in a first branchial cleft cyst or a dermoid cyst.
The anatomical location is the key to the diagnosis. Dermoid cysts are typically midline in location in the neck and first
branchial cleft cysts are located in the region of parotid gland, external auditory canal, and angle of mandible.
Lymphangiomas are typically located in the posterior triangle. On ultrasound, they appear multiloculated with
intervening thin septa. Abscesses appear as ill-defined, irregular collections with thick walls and internal debris.
Surrounding soft-tissue oedema, hyperaemia, and enlarged adjacent lymph nodes are also noted.

30
Ped Neuro
(CNS) 67. A 16-year-old female is referred for MRI after presenting with an increasing number of cutaneous
neurofibromata. As a child she had been noted to have a cafe-au-lait spot on her back. What MRI finding would
confirm the diagnosis of NF-1?
A. Multiple hyperintense white matter foci on T2WI.
B. Bilateral vestibular schwannomas.
C. Meningioma.
D. Optic nerve glioma.
E. Multiple ependymomas.

67. D. Optic nerve glioma.


NF-1 is an autosomal dominant neurocutaneous disorder. The gene locus is located on 17q11.2. Diagnosis of NF-1
requires two or more of the following: six or more cafe-au-lait spots, two or more neurofibromas (or one plexiform
neurofibroma), axillary/inguinal freckling, optic nerve glioma, sphenoid wing dysplasia or a first-degree relative with
NF-1. The classic imaging appearance is multiple focal areas of white matter and deep gray matter signal abnormality.
Other potential findings include intracranial stenoses and moyamoya type proliferation. NF-2 characteristically presents
with multiple intracranial schwannomas, meningiomas and ependymomas (MISME). Bilateral vestibular schwannomas
is diagnostic of NF-2.

(CNS) 68. A 16-year-old male with a history of epilepsy is investigated via MRI. Axial T2WI demonstrates a
cystic space within the left frontal lobe isointense to CSF. This is causing local mass effect and there is adjacent
enlargement of the left lateral ventricle. What is the most likely diagnosis?
A. Porencephalic cyst.
B. Arachnoid cyst.
C. Schizencephaly.
D. Hydranencephaly.
E. Ependymal cyst.

68. A. Porencephalic cyst.


Porencephaly is a congenital/acquired cystic cavity within the brain parenchyma with adjacent enlargement of the lateral
ventricle. They develop in utero or early infancy. Arachnoid cysts are also CSF isointense, but are extra-axial, displacing
the brain away from the adjacent skull. Ependymal cysts are intraventricular and the surrounding brain is usually normal.
Schizencephaly is characterized by an intraparenchymal cleft extending from the ventricular surface to the brain surface
lined by gray matter. Hydranencephaly results from an early destructive process of the developing brain. The cranial
vault is CSF filled with absence of the cortical mantle and ventricles (water-bag brain). Death in infancy is typical.

71. Follow-up MRI is performed on a foetus of 26 weeks gestational age after ultrasound raised the suspicion of
agenesis of the corpus callosum (ACC). This subsequently confirms that the callosum is absent. What is the most
likely additional radiological finding?
A. None, isolated abnormality.
B. Parenchymal T2WI signal hypointensity.
C. Periventricular nodular heterotopia.
D. Dysplastic brainstem.
E. Delayed sulcation.

71. E. Delayed sulcation.


Signs of ACC include absence of the cavum septum pellucidum, colpocephaly, high-riding third ventricle, and widening
of the inter-hemispheric fissure. ACC is reported to be isolated in <10% on foetal MR imaging. Sulcation delay is
present in most foetuses with ACC (particularly those imaged at <30 weeks gestation), including those with good
neurodevelopmental outcome, implying a global white matter dysgenesis. Periventricular nodular heterotopia and
parenchymal T2WI signal hypointensity are usually seen in association with abnormal sulcal morphology. Associated
posterior fossa abnormalities are also common, with cerebellar hemispheric abnormalities seen more than abnormalities
of the vermis. Brainstem abnormalities typicallyoccur in association with a cerebellar abnormality.

31
Ped Neuro
(CNS) 71. A 62-year-old man is referred for MRI of brain after presenting with cognitive decline, gait apraxia,
and urinary incontinence. There is a preceding history of chronic headache. Ventriculomegaly is noted on initial
CT. Which of the following conventional MRI findings distinguishes aqueductal stenosis from normal pressure
hydrocephalus?
A. Periventricular T2WI hyperintensity.
B. Normal sulci.
C. Aqueductal flow void.
D. Funnel-shaped aqueduct.
E. Relatively normal calibre fourth ventricle.

71. D. Funnel-shaped aqueduct.


Aqueductal stenosis is a focal reduction in aqueduct size, which can be congenital or acquired. Stenosis occurs at the
level of the superior colliculi or at the intercollicular sulcus. The best diagnostic clue is a funnel-shaped aqueduct. There
is resultant ballooning of the lateral and third ventricles. The fourth ventricle is normal distal to the obstruction. The
most specific finding is lack of CSF flow through the aqueduct on phase contrast MRI. Other findings include thinning
of the corpus callosum and downward displacement of the internal cerebral veins and third ventricular floor. In older
patients it can present similarly to NPH. Imaging findings in NPH include ventriculomegaly (with relative sparing of
the fourth ventricle) out of proportion to sulcal enlargement, with normal hippocampi. The aqueductal ‘flow-void’ sign
reflects increased CSF velocity through the aqueduct, although this can be observed in normal individuals.
MasterPass1
(CNS) 26 A nine-year-old girl presented with nystagmus, ataxia and diplopia to outward gaze. On CT there was a mass
within the optic chiasm, which extended into the left optic nerve. The mass had poor patchy enhancement. A further
eccentrically enhancing lesion was identified in left occipital lobe. The most likely underlying condition is
a Tuberous sclerosis
b Neurofibromatosis type 1
c Neurofibromatosis type 2
d Von Hippel-Lindau syndrome
e Sturge-Weber syndrome

26 Answer B: Neurofibromatosis type 1


Involvement of the optic tracts is typically seen in NF-1. Histologically, most lesions are low-grade astrocytomas
although 20% of chiasmal gliomas may behave aggressively. Imaging is best on MRI where lesions are usually hypo to
isointense on Ti and hyperintense on T2 with variable contrast enhancement.
(CNS) 27 A middle-aged male had a CT following prolonged seizures which showed a rounded cystic lesion in the
cerebellum with an avidly enhancing mural nodule. Other imaging demonstrated bilateral renal cell carcinoma. At what
other site is he at risk of developing further cystic lesions?
a Lateral ventricles
b Insular cortex
C Corpus callosum
d Spinal cord
e Pons

27 Answer D: Spinal cord


The underlying condition is von Hippel-Lindau syndrome (VHL). Differentials for intra-axial cystic lesions in the
posterior fossa in adults include metastases, haemangioblastoma, lymphoma and lipoma. Most haemangioblastomas
occur sporadically while 10-20% occur in VHL. In VHL 75% of haemangioblastomas occur in the cerebellum and 25%
in the spinal cord. Appearances in the spinal cord are those of a syrinx-like cyst with an isointense nodule that enhances
strongly after contrast.
(CNS) 29 A newborn baby was noted to have a thoracolumbar myelomeningocele and upper limb spasticity. An MRI
scan revealed a small posterior fossa, low-lying tonsils, an elongated fourth ventricle, tectal beaking and partial agenesis
of the corpus callosum. What is the most likely diagnosis?
a Dandy-Walker syndrome
b Chiari I malformation
C Chiari II malformation
d Chiari III malformation
e Septo-optic dysplasia

32
Ped Neuro
29 Answer C: Chiari II malformation
Chiari II is the most common and serious complex of anomalies resulting from a small posterior fossa. Chiari I is not
associated with a myelomeningocele and is an isolated hindbrain anomaly without supratentorial abnormalities. Chiari
III is rare and thought to be unrelated to Chiari I and II. It is associated with occipital/ cervical meningomyelocele.
Dandy-Walker is characterised by a large posterior fossa, vermian anomalies and cystic dilatation of the fourth ventricle.
(CNS) 30 A 15-month-old boy was investigated for developmental delay. On clinical examination he was found to have
a cleft palate. On CT he had a large posterior fossa, agenesis of the vermis with a large cystic dilatation of the fourth
ventricle and agenesis of the corpus callosum. What is the most likely diagnosis?
a Chiari I malformation
b Schizencephaly
C Dandy-Walker malformation
d Septo-optic dysplasia
e Vein of Galen aneurysm

30 Answer C: Dandy-Walker malformation


Dandy-Walker malformations are characterised by a large posterior fossa, vermin anomalies and cystic dilatation of the
fourth ventricle filling the entire posterior fossa. Midline CNS anomalies are seen in >60%. Differentials include a
posterior fossa extra-axial cyst, arachnoid cyst and a mega cisterna magna.

(CNS) 31 An 11 year old was investigated for widespread cutaneous lesions. Imaging revealed ribbon ribs, tibial bowing
and a hypoplastic sphenoidal ala on the left. An MRI of the spine was also performed which showed a mid-thoracic
lesion along with some bone remodelling. What is this lesion most likely to be?
a Ependymoma
b Astrocytoma
C Neurofibroma
d Lipoma
e Dermoid cyst

31 Answer C: Neurofibroma
The condition described is NF-1. Abnormalities within the spine occur in about 60% of patients. Most of these are
secondary to a neurofibroma which maybe dumbbell shaped along an exiting nerve root. Lateral thoracic meningoceles
are also known to occur. Meningiomas and ependymomas are more common in NF-2.
(CNS) 32 A 32 year old was imaged following sudden onset right hemiplegia. A CT showed subtle atrophy of the left
occipital lobe and unusual gyral calcification. MRI demonstrated prominent pial and deep medullary veins in the region
with a prominent left choroid plexus. What is the underlying aetiology?
a Tuberous sclerosis
b Ataxia telangiectasia
C Meningiomatosis
d von Hippel-Lindau syndrome
e Sturge-Weber syndrome

32 Answer E: Sturge-Weber syndrome


Sturge-Weber syndrome is a sporadically occurring phakomatosis in which facial port wine naevi, leptomeningeal
venous angiomatosis and orbital manifestations are described. Presentation is usually with seizures contralateral to the
site of facial naevus. The underlying aetiology is probably an abnormality in the development of cortical venous
drainage. Cortical atrophy, tram track gyral Ca, enhancing pial angioma and prominent draining veins may be seen.
(GIT) A patient with Von Hippel-Lindau syndrome was being investigated for renal symptoms and an incidental finding
in the pancreas was made on CT A 5-cm lobulated mass with the appearance of a `bunch of grapes' was seen in the neck
of the gland. There was a prominent central stellate scar within this lesion and distal dilatation of the pancreatic duct.
What is the most likely diagnosis?
a Acinar cell carcinoma
b Intraductal papillary mucinous tumour
c Papillary neoplasm of the pancreas
d Pancreatic ductal adenocarcinoma
e Serous cystadenoma of the pancreas

33
Ped Neuro
Answer E: Serous cystadenoma of the pancreas
A serous cystadenoma of the pancreas is a benign lobulated neoplasm composed of innumerable small cysts 1-20 mm.
The mass is usually lobulated with a mean size of 5 cm. Any part of the pancreas can be affected but it has slight
predominance for the head and neck. The pancreatic duct can be displaced, encased or obstructed. A prominent central
stellate scar is characteristic and there is a known association with Von Hippel-Lindau syndrome.

34 A 10-month-old child is admitted to intensive care and ventilated following a significant head injury. An unenhanced
CT brain is performed. Which of the following findings would be consistent with severe hypoxic brain injury?
a Decreased grey and white matter density, decreased grey/white matter differentiation and increased density of the
basal ganglia, thalami and cerebellum
b Increased grey and white matter density, decreased grey/white matter differentiation and decreased density of the basal
ganglia, thalami and cerebellum
c Increased grey matter density and decreased white matter density
d Decreased grey matter density, increased white matter density and decreased density of the basal ganglia, thalami and
cerebellum
e Increased grey and white matter density, increased grey/white matter differentiation and decreased density of the basal
ganglia, thalami and cerebellum

34 Answer A: Decreased grey and white matter density, decreased grey/white matter differentiation and increased
density of the basal ganglia, thalami and cerebellum
The `reversal' sign occurs in hypoxic ischaemic cerebral injury. It carries a poor prognosis. There is a high association
with non-accidental injury, but it can also result from events such as significant accidental trauma, near drowning,
cardiac arrest, status asthmaticus and status epilepticus.
(CNS) 46 A patient with known tuberous sclerosis had a routine follow-up CT. A 3 x 2-cm partly calcified
heterogeneously enhancing lesion was seen at the level of the foramen of Monro. What is the most likely pathology?
a Colloid cyst
b Subependymal giant cell astrocytoma
c Intraventricular
d Meningioma
e Germinoma

46 Answer B: Subependymal giant cell astrocytoma


Fifteen per cent of patients with tuberous sclerosis develop subependymalastrocytomas. They typically occur at the
foramen of Monro and are usually a well-defined rounded mass with some calcification. They usually enhance uniformly
with contrast and can degrade to a high-grade astrocytoma. Ninety-five per cent of tuberous sclerosis patients have
subependymal hamartomas. These occur in the periventricular region, are isointense to white matter on Ti and calcified
on CT. Fifty-five per cent of patients have cortical tubers, which are high signal on T2-weighted imaging.
50 An MRI brain of an infant born at term with evidence of normal development demonstrates myelination of the
brainstem, cerebellum and both the anterior and posterior limbs of the internal capsule. Neither the splenium nor genu
of the corpus callosum appear myelinated. What is the likely age of the infant?
a 2 months
b 3 months
C 6 months
d 9 months
e 1 year

50 Answer A: 2 months
Myelination occurs through the infant's brain and peripheral nervous system from caudal to cranial and from posterior
to anterior. Myelination milestones:
• term birth - brainstem, cerebellum, posterior limb of the internal capsule
• 2 months - anterior limb of the internal capsule
• 3 month - splenium of corpus callosum
• 6 months - gems of corpus callosum.

34
Ped Neuro
(GU) 57 A 10-year-old boy is demonstrated on ultrasound to have bilateral multiple echogenic masses measuring up to
4cm in size. A renal CT shows predominantly low-density (-20 HU) masses with some heterogeneous enhancement
following intravenous contrast. What underlying condition is the patient most likely to have?
a Von Hippel-Lindau disease
b Tuberous sclerosis
c Neurofibromatosis type 2
d Gorlin syndrome
e Hurler syndrome

57 Answer B: Tuberous sclerosis


The description is of multiple bilateral large angiomyolipomas, which occur in 80% of patients with tuberous sclerosis,
usually presenting by the age of 10 years. AML are also associated with neurofibromatosis type 1 and VHL disease.

MasterPass2
5 A 15-year-old boy presents with a lump on the left side of his neck, which has grown recently. It lies along the anterior
border of the sternocleidomastoid muscle. On ultrasound the lump is seen to be well circumscribed, compressible, have
internal structure but lack internal flow. The mass extends between the bifurcation of the internal and external carotid
artery. Using the Bailey classification, into which category would this lesion fall?
aI
b II
C III
d IV
eV

5 Answer C: III
The diagnosis is a second branchial cleft cyst. The internal structure is from haemorrhage or infection. The Bailey
classification is:
I Along anterior surface of the sternocleidomastoid, just deep to the platysma
II Along anterior surface of sternocleidomastoid, lateral to carotid space, posterior to submandibular gland and adherent
to vessels
III Extends medially between the internal and external carotid arteries to the lateral pharyngeal wall
IV Within pharyngeal mucosal space.
Type II is the most common.
(CNS) 9 A newborn baby boy developed respiratory distress post delivery. He was intubated and did well but as soon
as he was extubated he again developed respiratory distress. CT demonstrated bilateral air/fluid levels within the nasal
cavities and narrowing of the posterior choanae with bowing of the posterior maxilla. What is the most likely diagnosis?
a Polyp disease
b Unilateral choanal atresia
c Sudden infant death syndrome
d Encephalocele
e Bilateral choanal atresia
9 Answer E: Bilateral choanal atresia
Choanal atresia is failure of perforation of the oronasal membrane. Bilateral choanal atresia is a paediatric emergency
because babies are obligate nasal breathers until two to six months. The septation is either bony (85%) or membranous
(15 %). The bilateral form is slightly more common. It is associated with other congenital anomalies such as malrotation,
DiGeorge syndrome and foetal alcohol syndrome. With bilateral disease babies are intubated and endoscopic perforation
is performed.

(CNS) 20 A 10-year-old girl presented with a unilateral enlarged orbit. Facial X-rays revealed opacification of the
maxillary sinus with no air/fluid level. CT demonstrated depression of the right orbital floor with enlargement of the
orbit and lateral displacement of the lateral wall of the nasal fossa. The middle turbinate was large but morphologically
normal. What is the most likely diagnosis?
a Sinus hypoplasia
b Maxillary dentigerous cyst
C Primordial dentigerous cyst
d Acute sinusitis
e Ameloblastoma

35
Ped Neuro
20 Answer A: Sinus hypoplasia
All the conditions can cause opacification of the maxillary sinus. Sinus hypoplasia is the most likely answer given the
CT features. Maxillary dentigerous cyst contains a tooth or crown whereas primordial dentigerous cyst does not. In acute
sinusitis one would expect an air/fluid level in radiological investigations representing the disease process. Classical CT
findings of ameloblastoma are of multilocular lesion with scalloped borders on a background of diffuse groundglass
changes within the bone.

(CNS) 25 A nine-year-old boy underwent an MRI while being investigated for cognitive difficulties. Prominent occipital
horns and a high-riding third ventricle were seen. Sagittal sections revealed a radial orientation of the gyri. What is the
most likely diagnosis?
a Joubert syndrome
b Septo-optic dysplasia
C Corpus callosum agenesis
d Holoprosencephaly
e Porencephaly

25 Answer C: Corpus callosum agenesis


Agenesis of the corpus callosum can be partial or complete. The rostrum and splenium are absent or hypoplastic in the
partial variety. Widely separated lateral ventricles, with longitudinal white matter mater tracts (Probst bundles) indenting
the medial margins of the lateral ventricles and dilated occipital horns (col- pocephaly) are seen. Associated
abnormalities include Chiari II, Dandy-Walker malformation, migration disorders and lipomas.

(CNS) 26 A six-month-old male baby underwent neurological work-up. An MRI showed a wide CSF communication
between the left lateral ventricle and the cortical subarachnoid space. A similar smaller communication was seen on the
right without pouching along the lateral border of the right lateral ventricle. What is the most likely diagnosis?
a Schizencephaly
b Hydranencephaly
c Megalencephaly
d Lobar holoprosencephaly
e Arrhinencephaly

26 Answer A: Schizencephaly
Schizencephaly describes a full thickness CSF cleft lined by grey matter extending from the subarachnoid space to the
subependyma of the lateral ventricle. This can be either open lipped (the walls of the cleft are separated) or closed lipped
(the walls are opposed). The clefts may be unilateral or bilateral and asymmetric. The important differential is
porencephalic cyst, which results from insult to a normally developed brain.

(CNS) 27 A five year old with seizures and cognitive impairment had an MRI scan. This revealed features highly
suggestive of heterotopia. What are the likely findings on the MRI?
a CSF lined cleft extending from the ependymal surface to cortical pia
b Shallow Sylvian fissures and agyric cortex
c Bilateral nodular subependymal grey matter
d Squared appearance of the frontal horns and an absent septum pellucidum
e Poor brain sulcation with intraparenchymal calcification

27 Answer C: Bilateral nodular subependymal grey matter


Heterotopic grey matter occurs secondary to developmental arrest of migrating neuroblasts from the ventricular walls
to the surface of the brain. Nodular and laminar forms are described. Signal is isointense to grey matter on all sequences.
(CNS) 29 A healthcare worker noted that a three-month-old child had an unusual shaped head. She was seen by the
paediatrician who requested a CT head. This demonstrated bony fusion of the sagittal suture. The metopic and coronal
sutures were open. What is the most likely diagnosis?
a Brachycephaly
b Clover leaf skull
c Plagiocephaly
d Trigonocephaly
e Scaphocephaly

36
Ped Neuro
29 Answer E: Scaphocephaly
Craniostenosis is the premature closure of sutures. At birth all sutures are normally open. Typically, only one suture is
fused but in a quarter of cases more than one fuses. Early fusion results in an abnormally shaped head. Boys are more
commonly affected and it may be part of a syndrome. Scaphocephaly is the premature fusion of the sagittal suture, is
the most common and leads to a long thin head. Brachycephaly is premature fusion of the lambdoid or coronal suture
producing a short wide head. Plagiocephaly is unilateral fusion of the coronal and lambdoid suture producing a lopsided
skull. Trigonocephaly is the fusion of the metopic suture producing a forward-pointing skull. A cloverleaf skull typically
occurs in thanatophoric dysplasia and is due to premature closure of the sagittal, coronal and lambdoid suture.

(CNS) 30 A five-month-old child was being investigated for a nasopharyngeal mass. He had a cleft palate but otherwise
appeared morphologically normal. On CT there was a bony defect in the roof of the sphenoid sinus and a soft tissue and
CSF density mass extended into the nasopharynx. What is the diagnosis?
a Mucocele
b Transsphenoidal meningocele
C Transsphenoidal encephalocele
d Leptomeningeal cyst
e Dermoid cyst

30 Answer C: Transsphenoidal encephalocele


This patient has a transsphenoidal or basal encephalocele. Basal encephaloceles account for 10% of all encephaloceles.
They typically present with a soft-tissue mass within the nasal cavity, which can increase in size during the Valsalva
manoeuvre. Affected patients may be obligate mouth breathers due to nasal obstruction. It is associated with agenesis
of the corpus callosum, pituitary and hypothalamic dysfunction and hypoplasia of the optic nerves.

(CNS) 31 A 19-year-old boy was admitted with new onset seizures. On T2-weighted MRI there was high signal within
the atrophied left hippocampus. What other feature is likely to be associated with this condition?
a Enlargement of the occipital horn of the lateral ventricle
b Generalised cerebral atrophy
C Ipsilateral atrophy of the mammillary body
d Atrophy of the amygdala
e Heterotopia

31 Answer D: Atrophy of the amygdala


This boy has mesial temporal sclerosis, which is characterised by atrophy of the hippocampus, and high signal on T2-
weighted MRI. There is also loss of the normal interdigitation of the hippocampal head. It is associated with atrophy of
the ipsilateral mammillary bodies and fornix.

MasterPass3
6 You are approached by a paediatric registrar who has an externally performed cranial ultrasound report. The ultrasound
was performed in a district general hospital prior to the child being transferred to your hospital's neonatal intensive care
unit. The report concludes the child has a Grade 3 haemorrhage. What does this mean?
a Intraventricular haemorrhage with ventricular dilatation, 20% mortality
b Intraventricular haemorrhage with ventricular dilatation, 50% mortality
c Intraventricular haemorrhage without ventricular dilatation, 20% mortality
d Intraparenchymal haemorrhage, 70% mortality
e Intraventricular haemorrhage without ventricular dilatation, 10% mortality

6 Answer A: Intraventricular haemorrhage with ventricular dilatation, 20% mortality The grading system is:
1 Subependymal haemorrhage, no long-term abnormality
2 Intraventricular haemorrhage without ventricular dilatation, 10% mortality
3 Intraventricular haemorrhage with ventricular dilatation, 20% mortality
4 Intraparenchymal haemorrhage, 50% mortality.

37
Ped Neuro
11 A seven-year-old boy with developmental delay is being investigated for myoclonic seizures, which appear to be
reducing in frequency. On examination he has a facial angiofibroma. A CT brain is performed, which shows multiple
areas of cortical abnormality with a hypodense centre, broadened gyri and curved linear calcifications. Which other
intracranial abnormality may be seen?
a Giant cell astrocytoma
b Hydrocephalus
c Arteriovenous malformations
d Neuroblastoma
e Venous angiomas

11 Answer A: Giant cell astrocytoma


The diagnosis is tuberous sclerosis. It usually presents with a classic triad of facial angiofibroma, epileptic seizures
(these are usually the first sign but decrease in frequency with age) and mental retardation. There are a number of
associated CNS abnormalities: subependymal hamartomas, giant cell astrocytomas, cortical/ subcortical tubers and
heterotopic grey matter islands in the white matter.
19 A patient with known Dandy-Walker malformation presents with seizures and is investigated with a CT scan of the
brain. This demonstrates the large posterior fossa cyst expected in Dandy-Walker malformation, but also a high and
enlarged third ventricle with parallel lateral ventricles. The anterior horns of the ventricles are small in comparison to
the posterior horns. What other abnormality is represented on this scan?
a Prominent cavum vergae
b Hydrocephalus
C Holoprosencephaly
d Arachnoid cyst in the midline
e Agenesis of the corpus callosum

19 Answer E: Agenesis of the corpus callosum


Dandy-Walker malformation is characterised by an enlarged posterior fossa with a high tentorium, agenesis or
hypoplastic cerebellar vermis and cystic dilatation of the fourth ventricle. It is associated with other CNS abnormalities:
20-25% have dysgenesis or agenesis of the corpus callosum, 25% have holoprosencephaly and 25% have malformation
of the cerebral gyri.
(CNS) 25 A one-month-old hypotonic baby was morphologically abnormal. He had a cleft lip and hypotelorism. A CT
demonstrated fused thalami, a large monoventricle, agenesis of the corpus callosum, absence of the falx cerebri and
interhemispheric fissure. What further structure is most likely to be absent?
a Pituitary
b Septum pellucidum
C Septum vergae
d Olfactory nerves
e Optic nerves

25 Answer B: Septum pellucidum


This patient has holoprosencephaly, which is always associated with an absent septum pellucidum. Holoprosencephaly
is a failure or incomplete cleavage of the cerebral hemispheres. There are three types:
➢alobar - the most severe form, which typically results in death below one year of age. There is a large single ventricle
with a peripheral layer of cerebral cortex. No third ventricle, falx cerebri, interhemispheric fissure, corpus callosum,
septum pellucidum, olfactory or optic nerves are present. The thalami are fused
➢semilobar - is a milder form and children may reach adulthood. There is a single ventricle with partially formed
occipital horns and minimally developed temporal horns. There is partial fusion of the thalami, a small third ventricle
and rudimentary falx cerebri
➢lobar - mildest form where there is separation of the cerebral hemispheres. There are two cerebral hemispheres and
2 lateral ventricles. The frontal lobes are dysplastic and there is a single fused frontal horn. The thalami are separated.
(CNS) 26 A 14-year-old boy with recurrent fits since birth undergoes an MRI. On T2-weighted images there is gyriform,
low-signal intensity changes with associated atrophy affecting the left occipital lobe. On T1-weighted contrast enhanced
images there is enhancement of the leptomeninges. What other feature is likely on MRI?
a Right VI venous angiomatous lesion
b Left V2 venous angiomatous lesion
C Left occipital skull vault thinning
d Ipsilateral enlargement of the choroid plexus
e Cavernoma

38
Ped Neuro

26 Answer D: Ipsilateral enlargement of the choroid plexus


This child has Sturge-Weber-Dimitri syndrome, which is associated with a facial port wine stain (telangiectasia of the
trigeminal nerve) and a leptomeningeal venous angioma. The leptomeningeal venous angioma is most commonly in the
parietal lobe. It is associated with atrophy of the underlying cortex, thickening of the overlying skull vault and
enlargement of the choroid plexus. There is gyriform cortical calcification, which can sometimes be seen on plain film.
Angiomas may be seen in most visceral organs of the body.

(CNS) 27 A 15-month-old child was being investigated for chronic fits. On Ti- and T2 -weighted MRI a grey matter
cleft extends from the surface of the right temporal horn of the lateral ventricle to the parietal lobe cortex. CSF signal is
seen within this cleft. The corpus callosum is present but the septum pellucidum is absent. What is the diagnosis?
a Open-lipped schizencephaly
b Lobar holoprosencephaly
C Porencephaly cyst
d Closed-lipped schizencephaly
e Burr hole track

27 Answer A: Open-lipped schizencephaly


Schizencephaly is a cleft extending from the lateral ventricle to the cortex that is lined with grey matter. There are two
types of schizencephaly: open-lipped and closed-lip. Closed-lip describes the apposition of the two sides of the cleft
while in open lip there is CSF separating the two sides. Schizencephaly is caused by abnormal neuronal migration due
to an in utero ischaemic insult in the germinal matrix at between 30 and 60 days' gestation. Ninety per cent of
schizencephaly is associated with an absent septum pellucidum. It can also be associated with abnormalities to the optic
nerve in septo-optic dysplasia.

(CNS) 29 A premature baby girl born at 28 weeks' gestation was intubated for respiratory distress. On day three a cranial
USS showed right periventricular flare and hydrocephalus. A month later, on a repeat cranial USS, there was diffuse
cystic changes in the right periventricular region with dilatation of the right lateral horn. What is the most likely
diagnosis?
a Periventricular leukomalacia
b Porencephalic cyst
c Intraventricular haemorrhage
d Encephalitis
e Meningitis

29 Answer A: Periventricular leukomalacia


Risk factors for germinal matrix haemorrhage are prematurity (less than 3 2 weeks), low birth weight, males, multiple
births, prolonged labour and cyanotic heart disease. It typically occurs within the first two days of birth. The germinal
matrix lies in the caudothalamic groove and is very metabolically active which makes it sensitive to low levels of oxygen.
There are four grades of intraventricular haemorrhage as shown in the table below. A common finding in premature
babies is flare within the periventricular white matter. Periventricular leukomalacia is the sequelae of ischaemia in the
watershed areas, eventually the cystic areas are reabsorbed and ventriculomegaly occurs.

(GU) 29 A 37-year-old pregnant woman has been found to have a raised alpha-fetoprotein level and an ultrasound
anomaly scan is performed. The appearance of the foetus is abnormal with a flat inwardly scalloped contour of both
frontal bones and a posterior curve of the cerebellum. What is the most likely diagnosis?
a Holoprosencephaly
b Lissencephaly
C Choroid plexus cyst
d Vein of Galen aneurysm
e Spina bifida

39
Ped Neuro
29 Answer E: Spina bifida
Spina bifida is associated with Arnold-Chiari malformation in 90% and the ultrasound findings of Arnold-Chiari
malformation include hydrocephalus, abnormally pointed frontoparietal region (lemon sign) and abnormally shaped
cerebellum (banana sign).
(CNS) 30 A 23-year-old woman was undergoing an anomaly scan at 16 weeks' gestation. Her maternal alpha fetoprotein
level was very high. An obstetric USS demonstrated the banana sign and the lemon sign. What is the most likely cause?
a Schizencephaly
b Holoprosencephaly
C Meningitis
d Neural tube defect
e Septo-optic dysplasia

30 Answer D: Neural tube defect


The banana and lemon sign are seen on ultrasound in patients with neural tube defects. The head appears lemon shaped
at the level of the lateral ventricles due to bilateral indentation of the frontal lobe cortex, which is typically seen before
24 weeks. The banana sign is obliteration of the posterior fossa and herniation of the cerebellum.
32 A 16-month-old girl presented with an enlarging head circumference and bulging occiput. She was noted to have
poor fine motor control. An MRI of the head was performed, which showed a large uniformly low signal cystic area in
the posterior fossa on T1-weighted images and an elevated tentorium cerebelli. On axial images the cerebellar
hemispheres appeared widely spaced. What is the most likely diagnosis?
a Dandy-Walker malformation
b Large arachnoid cyst
C Cystic cerebellar astrocytoma
d Epidermoid cyst
e Haemangioblastoma

32 Answer A: Dandy-Walker malformation


Dandy-Walker malformation is characterised by an enlarged posterior fossa with a high tentorium, agenesis or abnormal
cerebellar vermis and cystic dilatation of the fourth ventricle. An arachnoid cyst is an ultrasound differential for Dandy-
Walker but the anatomy and source of the cystic structure should be evident on CT. This child is too young and
displaying incorrect symptoms for a diagnosis of epidermoid cyst. The cyst fluid in an astrocytoma is denser than CSF
and shows some degree of solid tumour thus allowing differentiation.

(CNS) 42 A 15-month-old child was placed under the care of social services for suspected child abuse. She had had a
CT scan aged six months, which was normal. A repeat CT scan demonstrated a defect in the right temporal bone with
indistinct scalloped margins and a prominent CSF space lying adjacent to it. There was right temporal encephalomalacia.
What is the likely diagnosis?
a Dermoid cyst
b Lacunar skull
C New fracture
d Leptomeningeal cyst
e Accessory suture

42 Answer D: Leptomeningeal cyst


A leptomeningeal cyst or `growing fracture' occurs in 1 % of all paediatric skull fractures. It is seen in fractures
associated with dural tears where arachnoid herniation and CSF pulsations produce fracture diastasis. Typical
appearance is of a skull defect with indistinct scalloped bony margins usually evident two to three months after the
injury. Gliosis of the adjacent brain parenchyma is common.

(CNS) 44 A three-month-old baby presents with intractable crying and respiratory distress. The patient has a facial CT
scan, which shows a bony septum extending across the posterior choanae. Which of the following additional features
are likely to be seen?
a 5-mm-wide posterior choanae
b Outward bowing of the posterior maxilla
C Thickening of the vomer
d Absence of the vomer
e 1-cm-wide posterior choanae

40
Ped Neuro
44 Answer C: Thickening of the vomer
The diagnosis is choanal atresia, which is the commonest cause of neonatal nasal obstruction. This is a life-threatening
condition, which is more commonly bilateral. It usually presents with respiratory distress, as babies are obligate nasal
breathers. Bony septations are present in 85-90% of cases, the remainder being membranous septations. Further imaging
findings include inward bowing of the posterior maxilla and narrowing of the posterior choanae to <3.4mm in a child
less than two years.

(GU) 47 A 15-year-old male presented with flank discomfort and was found to have renal cysts. What additional finding
would be diagnostic of Von HippelLindau (VHL) syndrome in this patient?
a A single pancreatic cyst
b Multiple pancreatic cysts
c Renal cell carcinoma
d A single central nervous system haemangioblastoma
e A phaeochromocytoma

47 Answer D: A single central nervous system haemangioblastoma


Criteria for diagnosis of VHL disease include: (1) more than one central nervous system haemangioblastoma, (2) one
central nervous system haemangioblastoma and any visceral manifestations of VHL disease, and (3) any manifestation
and a family history of VHL disease.
51 An MRI brain was performed on a toddler who had failed to attain gross motor skills after initially achieving them
on time. It was also noted that the child's head circumference was increasing faster than expected. The child's medical
history and birth record were unremarkable. The MRI showed communicating hydrocephalus. What is the most likely
underlying cause?
a Medulloblastoma
b Subdural haematoma
C Meningitis
d Venous obstruction
e Repetitive subarachnoid microhaemorrhage

51 Answer E. Repetitive subarachnoid microhaemorrhage


The most common cause of communicating hydrocephalus is repetitive subarachnoid microhaemorrhage mainly due to
repetitive trauma. The other causes are less frequent but can cause hydrocephalus.

(CNS) 54 A22 year old with widespread cutaneous lesions, axillary freckling and multiple cafe an laic spots is known
to have a neurocutaneous syndrome. What abnormality would you be most likely see on CT?
a Optic nerve glioma
b Acoustic schwannoma
C Haemangioblastoma
d Subependymal astrocytoma
e Subependymal nodules

54 Answer A: Optic nerve glioma


Tuberous sclerosis, neurofibromatosis one and two, von Hippel-Lindau, SturgeWeber and hereditary haemorrhagic
telangiectasia are all neurocutaneous disorders. The cutaneous lesions suggest this patient has neurofibromatosis type
one (NF1). Other cutaneous manifestations of NF1 are neurofibromas and iris hamartomas. Optic nerve gliomas are
seen in up to 30% of 1-M. In 1-F2 the main cutaneous finding is cafe an laic spots. In patients with tuberous sclerosis
there is adenoma sebaceum, shagreen patches and subungual fibrosis. In hereditary haemorrhagic telangiectasia there
are cutaneous telangiectasia on the face and freckles on the lips. A port wine stain is seen in the distribution of a branch
of the trigeminal nerve in Sturge-Weber.
(CNS) 57 A six-year-old girl with a repaired myelomeningocele at birth and hydrocephalus had an MRI scan to check
the position of her V-P shunt. Sagittal T1 and T2 images demonstrated a small posterior fossa with tectal beaking and a
cervicomedullary kink. What finding in the cervical spine would account for any recent onset of neurological symptoms?
a Cervical astrocytoma
b Syringomyelia
C Epidural abscess
d Neurofibroma
e Vertebral collapse

41
Ped Neuro
57 Answer B: Syringomyelia
The condition described is Chiari II. This is associated with syringomyelia in 20% and hydromyelia (50%). Hydromyelia
is distension of the central canal, while syringomyelia is CSF dissection through the ependymal lining to form a
paracentral cavity. Imaging appearances are indistinguishable and the entity is often grouped together as
syringohydromyelia. Syrinxes also occur in Chiari I and can be associated with spinal trauma, intramedullary tumours
and extramedullary compressive lesions.
Gupta
(CNS) 6. A 30-year-old man with a bimalar rash and learning difficulties was shown to have bilateral renal
angiomyolipomas. The most likely diagnosis is?
(a) Tuberous sclerosis
(b) Peutz-Jeghers syndrome
(c) Sturge-Weber syndrome
(d) Neurofibromatosis
(e) Fibrous dysplasia

(a) Tuberous sclerosis


The classical triad is adenoma sebaceum, mental retardation and seizures. Patients also commonly have bilateral renal
angiomyolipomas.

7. A 15-year-old student presents with history of seizures. CT shows multiple cortical and sub cortical calcified
lesions. Gadolinium-enhanced MRI of the head shows multiple enhancing masses in the subependymal regions. A
contrast-enhanced CT of abdomen shows multiple low-density masses in the liver and a large mixed attenuation mass
lesion in right kidney. The most likely diagnosis is?
(a) Sturge-Weber syndrome
(b) Tuberous sclerosis
(c) Sarcoidosis
(d) Klippel-Trenaunay syndrome
(e) Neurofibromatosis type 2

7. (b) Tuberous sclerosis


This is one of the phacomatoses with the classical triad of seizures, adenoma sebaceum and mental retardation. Other
findings include hamartoma of kidney (angiomyolipomas), heart (rhabdomyoma) and brain (tubers). These CNS
findings are typical and tubers are seen in the subependymal region, subcortical white matter and cortex. These
commonly calcify.
Sturge-Weber syndrome is characterised by multiple angiomatosis in face, eyes and leptomeninges. Sarcoidosis often
affects the meninges, peripheral nerves and patients may have multiple sclerosis like symptoms. Klippel-Trenaunay
syndrome presents with port-wine naevus, gigantism and varicose veins in affected limb. Neurofibromatosis type 2 is
characterised by bilateral schwannomas, meningiomas and ependymomas.

(CNS) 9. A 15-year-old boy presents with a history of epilepsy and visual loss. A CT scan shows ‘tram track’ gyriform
cortical calcifications in the right parieto-occipital lobe. MRI shows cortical atrophy in the region of calcifications. Post-
gadolinium T1 demonstrate focally enhancing leptomeninges and enlarged ipsilateral choroid plexus in the occipital
horn. What is the most likely underlying condition?
(a) Tuberous sclerosis
(b) von Hippel-Lindau
(c) Klippel-Trenaunay syndrome
(d) Sturge-Weber syndrome
(e) Neurofibromatosis

9. (d) Sturge-Weber syndrome


This is characterised by angiomatosis in the meninges, face and eyes. Meningeal angiomas result in cortical atrophy
underneath due to hypoxia. Calcifications are seen in the underlying gyri and ipsilateral choroid plexus thickening is
also seen. Retinal angiomas can lead to retinal detachment.

42
Ped Neuro
16. A neonate born with a history of prolonged labour has a routine cranial ultrasound which shows dilated lateral
ventricles. A subsequent MRI of brain and spine is performed which show a small posterior fossa, herniated cerebellar
tonsils through foramen magnum with hydrocephalus. The tectum has a beaked appearance. In the spine, there is a
myelomeningocele at the lower lumbar spine. What is the most likely diagnosis?
(a) Chiari type I malformation
(b) Chiari type II malformation
(c) Alobar holoprosencephaly
(d) Hydranencephaly
(e) Dandy-Walker malformation

16. (b) Chiari type II malformation


There is displacement of the fourth ventricle, brainstem and cerebellum into the cervical spinal canal and it is almost
always associated with myelomeningocele. Other findings may include beaked tectum, fenestration of falx,
hydrocephalus, colpocephaly and dysgenesis of corpus callosum. Chiari type I may show benign cerebellar ectopia up
to 5 mm below foramen magnum. There is no hydrocephalus or meningomyelocele. Alobar holoprosencephaly shows
a large single ventricle without occipital or temporal horns. There is no hemispheric development of the brain.
Hydranencephaly represents liquefaction of cerebral hemispheres, which are replaced with cerebrospinal fluid,
leptomeninges sac and remnants of cortex. Dandy-Walker malformation is characterised by an enlarged posterior fossa,
dysgenesis of cerebellar vermis and dilatation of 3rd ventricle.

(CNS) 16. A 27-year-old woman presents to the Accident & Emergency Department with headaches. A CT scan of the
head shows widely spaced lateral ventricles, dilatation of the trigones and occipital horns of lateral ventricles with an
upward displacement of the dilated 3rd ventricle. The underlying abnormality in the brain is?
(a) Midline arachnoid cyst
(b) Agenesis of the corpus callosum
(c) Prominent cavum septum pellucidum
(d) Hydrocephalus
(e) Lobar holoprosencephaly

16. (b) Agenesis of corpus callosum


This is associated with parallel, widely spaced lateral ventricles that may appear crescent shaped. There is dilatation of
trigones and the occipital horn of lateral ventricles, along with a high riding 3rd ventricle. Callosal agenesis is associated
with Dandy-Walker syndrome, Chiari malformations and fetal alcohol syndrome.
(CNS) 22. A 30-year-old man presents with loss of sensation in his toes. MRI of the cervical spine shows cerebellar
ectopia of 4 mm below the foramen magnum and syringomyelia of the cervical cord. The most likely diagnosis is?
(a) Chiari I malformation
(b) Chiari II malformation
(c) Chiari III malformation
(d) Chiari IV malformation
(e) Dandy-Walker malformation

22. (a) Chiari I malformation


Chiari I malformation is characterised by cerebellar ectopia and is frequently an isolated hindbrain abnormality without
supratentorial abnormalities. 20-30% of cases are associated with syringomyelia.
(CNS) 23. A 13-year-old child with a history of perinatal infection presents with headaches and vomiting. CT of the
brain shows gross dilatation of the lateral and third ventricles with a normal 4th ventricle. No tumour masses are seen.
The most likely diagnosis is?
(a) Aqueduct stenosis
(b) Klippel-Feil syndrome
(c) Chiari I malformation
(d) Dandy-Walker malformation
(e) Neurofibromatosis

23. (a) Aqueduct stenosis


There are various causes for aqueductal stenosis, however the imaging features of this condition are dilated lateral and
third ventricles with a normal 4th ventricle. This is the most frequent cause of congenital hydrocephalus. Its aetiology
may be classified as post-inflammatory (commonest), congenital or neoplastic (rare).

43
Ped Neuro
24. A newborn baby born at home presents with fits. CT of the head shows a large posterior fossa with agenesis of
cerebellar vermis and cystic dilatation of the fourth ventricle, filling the entire posterior fossa. Most likely diagnosis is?
(a) Megacisterna magna
(b) Dandy-Walker malformation
(c) Large arachnoid cyst
(d) Chiari type 2 malformation
(e) Porencephaly

24. (b) Dandy-Walker malformation


These are the typical features seen in Dandy-Walker malformation.
(CNS) 29. A 9-year-old boy with inguinal freckling presents with visual problems and epilepsy. MRI shows
homogenous enhancement of bilaterally enlarged optic nerves. What is the most likely diagnosis?
(a) Neurofibromatosis type 1
(b) Neurofibromatosis type 2
(c) Tuberous sclerosis
(d) von Hippel-Lindau disease
(e) Sturge-Weber syndrome

29. (a) Neurofibromatosis type 1


This is the most common type of neurocutaneous disorder. Common lesions encountered include café au lait spots,
peripheral nerve neurofibromatosis, optic nerve glioma, iris hamartomas, axillary and inguinal freckling, bony
abnormalities, pseudoarthrosis, scoliosis and duct ectasias. It is also associated with malignancies including
astrocytoma, malignant nerve sheath tumours, Wilms’ tumour, Rhabdomyosarcoma, leukaemia, thyroid carcinoma and
pheochromocytoma.
(GU) 47. A 35-year-old man with a facial ‘port-wine stain’ and history of epilepsy presents with haematuria. Contrast-
enhanced CT abdomen shows vascular malformations in the kidney and spleen. What is the most likely diagnosis?
(a) von Hippel–Lindau disease
(b) Sturge–Weber–Dimitri syndrome
(c) Neurofbromatosis type 1
(d) Neurofbromatosis type 2
(e) Tuberous sclerosis

47. (b) Sturge–Weber–Dimitri syndrome


This is characterised by multiple vascular malformations in the face (‘port-wine stain’) and central nervous system
(leptomeningeal V.angiomas), orbital and visceral angiomatosis (intestine, kidneys, spleen, thyroid, pancreas, lungs).
49. Transcranial ultrasound of a preterm infant with feeding difficulties shows echogenic shadowing filling the lumen
of right lateral ventricle. There is also dilatation of the lateral ventricles. What is the most likely diagnosis?
(a) Normal choroid plexus
(b) Subependymal haemorrhage
(c) Subependymal haemorrhage with ventricular dilatation
(d) Subependymal haemorrhage without ventricular dilatation
(e) Periventricular haemorrhage

49. (c) Subependymal haemorrhage with ventricular dilatation


The DD is from a normal choroid plexus. However, if the blood fills the ventricle then the diagnosis is easy.
EDiR MCQ
15. The following are CNS features of tuberous sclerosis.
(a) Presentation is usually with seizures.
(b) Subependymal nodules are most common in the occipital horn of the lateral ventricles.
(c) Pilocytic astrocytoma is a complication.
(d) Cortical tubers are most prominent on T1W MRI.
(e) Calcification may be seen in upto 50 % on skull X-ray.

44
Ped Neuro
Answers:
(a) Correct
(b) Not correct
(c) Not correct
(d) Not correct
(e) Correct

Explanation: In tuberous sclerosis subependymal nodules are most common along ventricular surface of caudate
nucleus, with cortical tubers which are most prominent on T2W and FLAIR. Giant cell astrocytoma is a complication.
20. Which of the following are correct regarding Sturge-Weber syndrome
(a) Cortical gliosis is a feature.
(b) It is accompanied by lepto-meningeal angiomas on the contralateral side.
(c) Underlying cortical calcification is common.
(d) Angiomas are more common over the frontotemporal regions.
(e) It involves a port-wine stain affecting the trigeminal nerve distribution.
Answers:
(a) Correct
(b) Not correct
(c) Correct
(d) Not correct
(e) Correct
Explanation:
In Sturge-Weber syndrome, lepto-meningeal angiomas are seen on ipsilateral side and angiomas are more common
over parieto-occipital region.
24. Radiological features of absent corpus callosum include:
(a) A high riding third ventricle
(b) Enlargement of the occipital horns
(c) Crescentic lateral ventricles
(d) Hypoplasia of the optic nerves
(e) Separation of pericallosal arteries on angiography

Answers:
(a) Correct
(b) Correct
(c) Correct
(d) Correct
(e) Correct

Explanation:
Dysgenesis of the corpus callosum may be complete (agenesis) or partial and represents an in utero developmental
anomaly. There appears to be a male predilection (M:F ~2:1). Maternal alcohol consumption during pregnancy has been
recognized as another risk factor.
(GU) 26. Features of von Hippel-Lindau (VHL) disease include:
(a) Renal cysts are present in over 50%.
(b) Renal angiomas may be distinguished from renal cell carcinoma by imaging.
(c) Renal impairment is common.
(d) Renal cell carcinomas are usually solitary.
(e) A cyst with an enhancing nodule is suspicious for malignancy.

Answers:
(a) Correct
(b) Not correct
(c) Not correct
(d) Not correct
(e) Correct

45
Ped Neuro
Explanation:
Renal angiomas cannot be distinguished from renal cell carcinoma on imaging. In VHL renal impairment is uncommon
and renal cell carcinomas are usually bilateral and multicenteric.
27. Regarding the Dandy-Walker malformation which of the following are correct:
(a) There is inferior displacement of the vein of Galen
(b) There is a high lying tentorium
(c) The cerebellar vermis is normal
(d) The corpus callosum is absent in up to 25 %
(e) The posterior fossa is small

Answers:
(a) Not correct
(b) Correct
(c) Not correct
(d) Correct
(e) Not correct

Explanation:
Dandy Walker malformation is the most common posterior fossa malformation and consists of a triad of hypoplasia of
vermis with cephalad rotation of vermis remnant, cystic dilatation of fourth ventricle and enlarged posterior fossa. There
is superior displacement of vein of Galen.
6. Which of the following statements are correct about Chiari malformation:
(a) In normal physiology, cerebellar tonsils descend with increasing age.
(b) Cerebellar tonsils laying 3 mm below the foramen magnum is a normal finding in the second decade.
(c) Is usually associated with other congenital brain anomalies.
(d) Is rarely associated with spinal cord lesions.
(e) Is associated with Klippel-Feil anomaly (fused cervical vertebrae).

Answers:
(a) Not correct
(b) Correct
(c) Not correct
(d) Not correct
(e) Correct

Explanation:
In normal physiology, cerebellar tonsils ascend with increasing age. In Chiari 1, there is downward displacement of
tonsils below the foramen magnum by more than 5 mm. It is usually associated with spinal cord and skull base
abnormalities rather than other brain anomalies.
7. Which of the following statements are correct about Tuberous sclerosis:
(a) Cortical tubers enhance following intravenous contrast in the majority of cases on CT.
(b) Subependymal nodule enhancement indicates malignant transformation.
(c) Subependymal giant cell astrocytoma is seen in the 10-20 % of cases.
(d) Renal angiomyolipomas are seen in 5-10% of cases.
(e) Cardiac rhabdomyoma is a recognized manifestation.

Answers:
(a) Not correct
(b) Not correct
(c) Correct
(d) Not correct
(e) Correct
Explanation:
In tuberous sclerosis, the cortical tubers are seen in 95% of patients although enhancement is seen in less than 5% of
tubers. 30-80% of all subependymal nodules enhance after intravenous contrast but it does not indicate malignant
transformation. Renal Angiomyolipomas are seen in 40-80% of cases of tuberous sclerosis.

46

You might also like